Você está na página 1de 102

CIVIL PROCEDURE RESSLER FALL 2017

PERSONAL JURISDICTION
I. General Information:
a. Pennoyer v. Neff- Need to bring the person into the jurisdiction of the court at the outset of
the suit
i. Two Principles:
1. Every state possess exclusive jurisdiction over person and property within its
territory
2. No state can exercise direct jurisdiction and authority over persons and property
w/o its territory
b. PJ requires two things:
i. Statutory authorization (long-arm statutes)
ii. Constitutional authorization (minimum contacts)

II. Automatic Personal Jurisdiction:


a. Domiciliary of the state: Current dwelling and intent to stay
i. Must physically move there
b. Personally served within the state (Burnham)
i. Being in the state is considered sufficient minimum contact
ii. Bottom line: Judges all agree that service in state is sufficient for PJ—(just differ on
reasoning)
iii. Exception: Doesn’t work if the person was tricked into coming into the state OR if you
solely appear to be part of a case pending in that state (e.g. witness or special appearance)

c. Consent
i. Appoint an agent for service of process
ii. Contract with a forum selection clause
1. May be able to fight it if you can prove:
a. Forum selection clause deprives you of a meaningful day in court, OR
b. Prove lack of notice of the forum-selection clause

iii. Consent to abide by the court’s determination regarding PJ once you show up and contest
jurisdiction
d. Erroneous appearance
i. Appeared in the state erroneously, not under special service, and was served
ii. Come in for a different purpose like a traffic ticket and appear before the court
e. If yes to any of these question, you have PJ, if not look to the following items:
III. Step 1: Long Arm Statutes (Statutory Authorization) (Call D’s back into the state to defend
against a
lawsuit)
a. Due process doesn’t actually confer any jurisdiction on state courts; it only defines the outer
bounds of permissible jurisdictional power—it is up to the legislation of each state to actually
grant
their courts PJ
b. State long arm statutes-authorization to the courts of power to have jurisdiction over
anyone.
Three types:
i. Authorize jurisdiction to the maximum extent allowed by due process (CA)- Min. cont
test
ii. Enumerated list (Authorizes jurisdiction over D’s based on specific types of contact
w/forum state)
1. If this type, check statute first, then min. contacts
2. This is a literal construction interpretation (must fall within the list)
3. Tortuous acts: (Two interpretations)
a. Felt in the state = jurisdiction (Ill Rule)

Downloaded From OutlineDepot.com


b. Must be committed in the state = jurisdiction (NY Rule)

iii. Enumerated, but interpreted to be the extent of due process—Min contact test
c. Where PJ would be constitutional, but the long arm statute does not authorize it, some
courts
allow P’s to use quasi-in rem jurisdiction as an alternative statutory basis
d. Federal long arm statutes (Rule 4k)
i. 4(k)(1)(a)- Directs you to the state long arm statute
1. If state statute authorizes it, move on to minimum contacts
a. Really a three part test:
i. Meet 4(k)(1)(a)
ii. Meet the states long arm statute
iii. Must be constitutional—have min. contacts

ii. Can also use in rem as statutory authorization


1. In rem still requires minimum contact

IV. Step 2: Purposeful Availment- Minimum Contact Test Directed at the State (Constitutional
Authorization)
Two Parts
Part 1: Minimum Contacts (Moved away from “presence” and “consent”) States no longer need
an implied
“consent” statute like in Hess
NOTE: P does NOT have to meet in min. contact requirement—it is only for D
Part 2: Reasonableness/Fairness—Traditional notions of fair play and substantial justice
Five Factors

a. (1) Continuous or systematic (all of the contacts—regardless if related to the suit)


i. Specific Jurisdiction: Found when contacts are not necessarily continuous and systematic,
but are related to the action
ii. General Jurisdiction: Found when the contacts are so systematic and so continuous that a
forum has PJ over them on any action brought
1. If it is the corporation’s HQ, it will probably have general jurisdiction in that state
2. You have general jurisdiction in your domicile, you may have it if you are a resident
iii. TEST NOTE: Always mention that you are going to go for specific, if that is what you want;
otherwise, if you start making a lot of arguments about a lot of unrelated contacts, it looks
like you are arguing for general jurisdiction
b. (2) Relatedness (contacts just related to the suit)
i. Is it specifically related to the suit
c. (3) Foreseeability
i. Unilateral Act (WWVW)- If unilateral act of P—no jurisdiction
1. If D has a contact w/a 3rd party and the 3rd party unilaterally, w/o D’s control gets
contact w/the state, then D is NOT subject to suit based on the 3rd party’s contact
2. Corp. must reach out to make sales—if corp. puts product into the stream of
commerce expecting to sell into that state, than the state has PJ, but if consumer
brings the item there by a unilateral act—NO PJ

d. (4) Purposeful availment


i. Contracts (BK v. Rudzewicks)
ii. Goods: Stream of Commerce (Asahi v. Superior Court)
1. O’Connor- Stream of Commerce “Plus” (4 Votes)
a. Awareness of stream of commerce is NOT enough for purposeful availment
of minimal contacts
b. Need some additional activity aimed at the specific state for it to be enough
2. Brennan- Stream of Commerce (4 Votes)
a. Anything within the chain of distribution is within the stream of commerce
and therefore constitutes minimum contacts

iii. Property

Downloaded From OutlineDepot.com

1. Property is now only a contact—must be evaluated according to minimum contact


test
2. Recovery limited to amount of property attached
3. Only true and quasi 1 really will give you PJ
a. True In Rem: Jurisdiction over property (“Against the property”)
i. Will have “minimum contacts” b/c all contacts related to the
property in the state, subject to specific jurisdiction
ii. Gives specific jurisdiction
iii. Shaffer- Got rid of the “attach” req. and all you have to do is look at
the min. contacts to meet the constitutional requirement

b. Quasi In Rem 1- Dispute about the property


i. Determines rights and property against specific claimants
1. Adverse possession, mortgage v. mortgagee
ii. Gives specific jurisdiction
c. Quasi In Rem 2- Dispute is unrelated to the property OR property that is
related, but not about ownership
i. Must attach the property at the outset to assert the court’s
jurisdiction over the property
ii. This is a gap filler if the states long arm statute is crappy-P can
attach the property and use in rem to get PJ even when the states
long arm statute wouldn’t allow it

iv. Calder Test


1. What state is targeted in the article/website?
2. What state feels the “effects” of the article/website
3. Activities in article took place in forum
v. Internet- Zippo Test
1. How interactive is the website?
a. Passive site = No PJ
b. Interactive = Use Calder test to see if directed at people in the forum
2. How related is the internet contact to the related suit?
vi. Corporations
1. May be sued in states where they have purposeful contacts, though the state of
incorporation has significance (only for general jurisdiction)

e. (5) Fairness/Reasonableness (traditional notions of fair play and substantial justice)


(Memorize
all 5)
(1) Burden on the defendant
(2) Interest of the plaintiff in obtaining relief
(3) State interest in adjudicating the dispute
(4) Efficiency of the interstate judicial system
1. Where are the witnesses from, etc...
(5) Shared interest of several states in their substantive social policies
i. NOTE: These reasonableness tests come into play when min. cont. are really weak, IF min.
cont. are very strong, then the reasonableness test will ONLY matter if it creates a high level
of unreasonableness creating a constitutionally concern or it denies a person their day in
court
ii. Must be unconstitutionally unreasonable for it to really affect the situation

SERVICE & PROCESS


I. Two part process: Constitutional and Statutory tests
II. Notice: Constitutional Requirement
a. DP of 5th and 14th amendment require: 1) Reasonable notice AND 2) Opportunity to be
heard
Downloaded From OutlineDepot.com

b. DP requires notice that must “under all the circumstances, (1) be reasonably calculated to
apprise interested parties of the pendency of the action and (2) afford them an opportunity
to present their objections” (Mullane- merely published it in the newspaper)
i. If you have actual notice that they don’t know, you must provide them more information
c. Ordinarily, notice by publication is NOT sufficient
d. Must at least mail it to them when practical and you know their address
III. Service (FRCP 4): Statutory Requirement
a. Service requires two things 4(c)(1): 1) Summons and 2) Copy of the complaint
b. Time limit for service— Have 120 days to serve after filing the complaint – 4(m)
i. Fed Question: Statute of limitations (SOL) tolls when you file the suit
ii. Diversity Jurisdiction: SOL will start tolling by whatever the state statute says (two
possibilities)
1. File the suit
2. By service of suit
c. Who can serve process
i. 4c1- Plaintiff is responsible for service of summons and complaint within Rule 4(m)—120
days
ii. 4c2- Service may be performed by anyone (1) NOT a party to the suit and (2) at least 18
yr old
1. 4c3- At request of P, the court may direct that service be effected by a US Marshall

d. How to serve process on an individual within US judicial district—FRCP 4e


i. Service upon a person from whom a waiver has not been obtained and filed (other than an
infant or incompetent person may be effected in any US judicial district)
1. 4e1- pursuant to the law of the state in which the District Court is located, OR
2. 4e1- pursuant to the law of the state in which service is effected, OR
3. 4e2- delivering a copy of the summons and complaint to the individual personally,
OR
4. 4e2- by leaving copies at the individuals dwelling or place of abode with some
person of suitable age and discretion then residing thereon, OR
a. Suitable age and discretion is usually over 18 yrs old
b. General Rule: You must choose their main home to serve them (even if you
spend hundreds of millions building/creating a second home)

5. 4e2- by delivering a copy of the summons and complaint to an agent authorized by


appointment or by law to receive service of process
e. How to serve process on infants and incompetents—FRCP 4g
f. How to serve process on Corporations and associations—FRCP 4h
i. According to provisions in 4e1 (in accordance to state law where court sits), OR
ii. By delivering summons and complaint to an officer, manager, or general agent, OR
iii. By delivering the summons and complaint to an agent authorized by appointment to receive
such process and if by statute, by also mailing it to defendants

g. How to serve process on the US and its agencies, corporations, or offices—FRCP 4i (Must
serve
to at least two people and maybe even three)
i. Deliver a copy to one of three people: (NOTE: You cannot mail it!)
1. Attorney of the district where the suit is brought,
2. Assistant US attorney, OR
3. Clerical employee of the US
ii. Mail a copy to the attorney general of the US in Washington
iii. Send a copy to the agency or the officer
iv. NOTE: This rule requires duplicate service (usually 2-3 people)
h. In rem service—FRCP(n)(2)
i. You may use in rem to serve someone IF you prove that the other methods didn’t work

Downloaded From OutlineDepot.com


1. You MUST first show the other methods didn’t work

SUBJECT MATTER JURISDICTION


I. General Principles:
a. SMJ canNOT be CONSENTED to or WAIVED by either party
b. You can assert at any time that there is no SMJ, if correct, the case will be thrown out (even
after
verdict)
c. SMJ canNOT be waived by the courts
d. Types:
i. State Courts- Have general jurisdiction
1. The aggregate of state courts can hear pretty much anything (there are five
exceptions)
2. States are free to divide SMJ among whatever courts they decide
ii. Federal Courts- Have limited jurisdiction
1. Limited to what is contained in Article III—if not in Art. III, then there is no SMJ
2. Generally have two types:
a. Diversity jurisdiction
b. Federal Question jurisdiction

II. Federal Question Jurisdiction


a. Purposes of the rule are to procure the following in the interpretation of the law (USE):
i. Uniformity (Don’t want states making different laws concerning fundamental rights)
ii. Sympathy (Fed judges more sympathetic to policies underlying federal legislation)
iii. Expertise
b. §1331- District Courts have “original jurisdiction of all actions arising under the constitution,
laws,
or treaties of the US”
c. Holmes test: What law creates the question or cause of action
d. Well Pleaded complaint rule (Mottley): (§1331- “arising under” interpretation=well pleaded
complaint)
i. Federal issue must be in the complaint!—cannot be anticipated by the answer or by the
reply, but must be in the original claim
ii. Even if P knows that D is going to answer with federal law, it is NOT enough for P to get into
federal court
iii. Well-pleaded rule is NOT part of the constitution—congress could define what “arises
under” but they haven’t done that thus far

III. Diversity Jurisdiction


a. Purposes:
i. Protect an out-of-state litigant from in-state court prejudice
ii. Don’t want to waste federal courts resources deciding a state law question
b. Applies to both Ps and Ds-draw a line between Ps and Ds and none of the parties may be
from the
same state
c. Constitution doesn’t require complete diversity—that is only required by §1332(a)
i. General: Fed courts have jurisdiction when the matter exceeds $75,000 and is between...
1. Citizens of different states
2. Citizens of states and citizens of a foreign state
3. A foreign state as P and citizens of a state or of different states

d. Two requirements:
i. Jurisdictional amount OVER $75,000
1. Legal Certainty Test/Requirement:
a. To remand on this amount, it must appear to a legal certainty that P canNOT
get the amount claimed in the complaint (this typically will only occur if
damages are fixed by statute or claiming punitive for a contract claim)

Downloaded From OutlineDepot.com

2. Aggregation Rules:
a. If one P vs. one D, then P in such a case may aggregate all of her claims to
meet the jurisdictional requirement, even if the claims are unrelated legally
or transitionally (add only P’s up—not D’s claim)
b. If multiple parties on either side, then:
i. Cannot aggregate, unless D’s were joint and severally liable (joint
tortfeasors)
1. Meaning, either D could be completely liable for the full
amount

ii. Cannot aggregate multiple P’s claim even if arise out of same T/O,
unless a “common, undivided interest” in a single action exists
1. Ex: Plot of land and joint tenancy
iii. Personal injuries suffered by different people are separate claims
and cannot be aggregated

c. Counter claims CANNOT be aggregated to meet the threshold


3. Injunctive/Equitable Relief:
a. Amount is either (courts typically allow either)
i. Cost of the harm to P, OR
ii. Amount it would cost D to remedy the situation

ii. Complete Diversity- No jurisdiction if there is any overlap of citizenship between Ps and
Ds
1. If P is from state A and 99 Ds are from state B, but 1 D is from state A, then it will
have to be in state court

e. Citizenship of an individual: (Mas v. Perry) – Citizenship does NOT equal residency!


i. Citizenship of individual is their domicile
ii. Only way to change domicile is:
1. Change residence (must physically move there), AND
2. Intention to remain their permanently
iii. You may only have ONE domicile at a time
iv. Marrying an alien does NOT make you lose your US citizenship
f. Citizenship of an estate representative
i. Where the decedent was a citizen
g. Legal Representation for insane or infant
i. Where the insane or infant is a citizen
h. Citizenship of a corporation
i. Corporations may have two citizenships: (§1332)
ii. Citizen of every state of its incorporation (may be more than one), AND
iii. Citizen of one state in which it has its “principal place of business” (ONLY one test now)
1. Nerve Center test (Brain- e.g. Intangible production co’s, Microsoft):
a. When corps activities are far flung, the sole nerve center of that corp is more
significant in determining place of bus b/c that is where all decisions are
made

2. If corporation is incorporated in a foreign country, it is an alien for diversity


purposes
a. However, if its principal place of business in U.S., it is also a citizen in that
state
i. Citizenship of a partnership
i. That of each partner
j. Citizenship for non-incorporated businesses
i. The business is considered a citizen of all states of which its members are citizens
ii. If a labor union is big, it could have members in all states and thus no diversity jurisdiction
Downloaded From OutlineDepot.com

IV. Supplemental Jurisdiction


a. If P files a federal question and a state law claim, through supplemental, and if comes out of
the
“common nucleus of operative facts”, then supplemental jurisdiction allows the plaintiff to
bring
the state law claim in federal court even though there is no diversity jurisdiction
i. As long as federal question is a bona fide claim at the outset, if later the federal question is
thrown out, state claim is still adjudicated by the federal court

REMOVAL AND REMAND


I. General Principles:
a. Removal- Moving a case from state to federal court
b. Remand- Moving a case from federal to state court
c. The case must be one over which federal courts have SMJ for it to be removed
d. Only D can remove: P cannot remove if there is a counterclaim asserting a right under fed.
law
II. Removal Statutes: 28 USC §1441 & 1446
a. §1441(a)
i. Can remove any action brought in state court where the fed. court has original
jurisdiction—can remove only if P could have brought it in federal court

b. §1441(b)
i. If federal question—automatic removal
ii. If diversity, canNOT remove IF one of the Ds is a citizen of the state from which removal is
sought AND Can’t remove if you seek for LESS than $75,000

c. §1446- Procedure for removal


i. (a)- D files notice of removal in the District Court
1. Action may be removed by D or Ds—ALL D’s must agree to remove or it is NOT
removable

ii. (b)- After filing notice, D gives notice to all parties and files a copy of the notice with the
state court
iii. (c)- State will STOP proceedings, and all will immediately go to federal court
iv. (d)- D has 30 days to remove after being served with the complaint
1. If case becomes removable (one of the Ds dropped or fed question added), then you
have 30 days to remove from when case became removable
2. In diversity cases, have 1 year during which a case can become removable

III. Remand Procedures – 28 USC §1447(c)


a. Once removed by D, state court has NO say— P must go to fed court and contest the removal
b. P has 30 days to file a notice in federal court to remand the case back to state court UNLESS
he files
because of SMJ (then he has all the time P wants)
c. §1447(d)- An order remanding a case to state court from which it was removed is NOT
reviewable
on appeal
i. CAN appeal removal to federal court, where court has refused to remand the case and the
final judgment has been made

VENUE & FNC


I. General Principles:
a. Venue- Where within the district or state should the trial be held
b. Purposes
i. Protect D from burdensome trials
ii. Preserve efficiency of court systems
iii. Restrict to 1 or a few of all the possible courts in the state

II. State Courts


a. Transfer of venue is county to county—never from state to state
b. For a state to state transfer in a state court, you will use a forum non conveniens dismissal

Downloaded From OutlineDepot.com


III. Venue is proper for Diversity and Federal Question in...
a. Where any defendant resides if all defendants reside in the same state, OR
i. If there is only one district in the state, this will not be an issue
ii. If all Ds reside in CA, but all the Ds live in different districts within the state, you can file in
ANY of the four districts
1. Ex- If 99 D’s live in district 1, and only 1 D lives in district 2, P can still file in district
2

b. Where a substantial part of the events giving rise to the claim occurred or where a
substantial part
of the property that is the subject of the action is situated
c. Fall Back Provision: If options A or B do NOT apply and there is no district where it can be
brought, then venue will be proper where D is properly subject to PJ when the action
commenced
i. Only happens if the underlying cause of action occurs in a foreign country or all the different
parties reside in different states
d. An alien may be sued in ANY jurisdiction
IV. Where does D reside for venue purposes? (Based on residency—NOT citizenship!)
a. Individuals
i. Majority view: Residence = D’s domicile—have only one residence and that is your
citizenship
b. Corporations
i. A corporation is a resident in any jurisdiction that has PJ over it
ii. To discover residence, analyze minimum contacts with each judicial district as though
it were a state. If minimum contacts with that district are sufficient, venue is proper
iii. If no such district, then corporation shall be deemed to reside in the district within which it
has the most significant contacts

V. Change of Venue
a. §1404(a)- Proper Venue
i. Transfer of venue is proper if...
1. Convenience of the parties and witnesses, and
2. In the interests of justice, and
3. Must be to a district or division where it might have been brought
ii. Burden of proof on party seeking transfer
iii. Use law of transferor court (no change of law when case is transferred)
b. §1406(a)- Improper Venue
i. May be dismissed, or in the interest of justice transferred to any district or division where it
might have been brought
ii. Use law of transferee court (choice of law rules will change)
iii. Regardless if it is §1404 or §1406, they will use the choice of law in the transferee
court

c. NOTE: Ds CANNOT waive PJ in change of venue—they can in other situations, just not here
VI. Choice of Law
a. §1404(a)- Use law of transferor court
b. §1406(a)- Use law of transferee court
c. Klaxon rule: Federal courts, sitting in diversity cases, must use state law in which they sit

VII. Forum Non Conveniens (FNC)

a. Results in actual dismissal of the case, and it must be filed again for a more convenient forum
b. This was used before §1404 was around—now if you want to transfer to more convenient
federal
district ct, just use §1404 rather than dismiss with a FNC
c. Use FNC when in fed court and you think that a foreign court is more convenient
d. Use FNC when in state court and there is a more convenient court in another state
e. Choice of law does NOT apply—Meaning, FNC may change the law that is applied

Downloaded From OutlineDepot.com

f. No FNC if the court knows that the more convenient forum will not hear the case—must be
the
MOST convenient forum where can be subject to suit
g. Main focus: Is it convenient or not
h. Two step process:
(1) D must show that an adequate forum is available
i. Possible change in law is NOT enough to make it inadequate UNLESS the changes makes the
remedy “so clearly inadequate or unsatisfactory that it is no remedy at all”
(2) Must show considerations of party and forum override P’s choice of forum and justify
dismissal
ii. Two types of factors to consider:
1. Look at private factors to decide if FNC is proper:
a. Relative ease of access to sources of proof
b. Availability of witnesses—both compulsory process for attendance of
unwilling, and the cost of obtaining willing witnesses
c. Possibility of viewing the premises
d. All other practical problems that make trial of a case easy, expeditious, and
inexpensive

2. Look at public factors to decided if FNC is proper:


a. Administrative difficulties flowing from court congestion
b. Local interest in having localized controversies decided at home
c. Having a diversity case trial in a forum that is at home with the law that
must govern the action
d. Avoidance of conflict of law
e. Avoidance of application of foreign law
f. Unfairness of burdening citizens in an unrelated forum with jury duty

RAISING/CHALLENGING JURISDICTION CHALLENGES


I. Direct Attack
a. Appear in court to battle it out
b. Special Appearance
i. Only worked for PJ questions
ii. If you raised anything else besides PJ, it becomes a general appearance and you waived PJ
c. Most courts now follow FRCP 12 (This got rid of special and general appearances)
i. FRCP 12(b)- Things you may contest...
1. Lack of SMJ
2. Lack of PJ
3. Improper venue
4. Insufficient process
5. Insufficient service of process
6. Failure to state a claim upon which relief can be granted, and
7. Failure to join a party under Rule 19
ii. Must be raised in either:
1. Motion before responding, OR
2. Pleading—the answer itself
3. Note: Must include ALL of your defenses at once
iii. FRCP 12(h)(1)- A party waives any defense listed above in (2-5) by:
1. Omitting it from a motion when first filed, OR
2. Failing to make a motion or include it in a responsive pleading
iv. Summary of Timelines for Motions
1. 12(b)(2-5)- Must be asserted in the first response
2. 12(b)(6-7)- Must be asserted anytime before judgment
3. 12(b)(1) (SMJ)- Anytime (just not in collateral attack)

Downloaded From OutlineDepot.com

d. Good to do direct attack when you can win on merits alone or not liable
II. Collateral Attack
a. Do not appear—your attorney does nothing and they enter default judgment against you
b. ONLY thing you can contest is PJ
i. You CANNOT do collateral attack on SMJ
c. If your case has strong case on its merits, DON’T use collateral attack
d. If on collateral attack, the court decides that the initial judgment is void b/c of lack of PJ, then
the
suit can be re-filed but it will have to be filed in a different place (where PJ exists)

ERIE QUESTION- WHAT LAW APPLIES


I. Generally
a. Erie applies in diversity cases
b. Is the state law substantive or procedural?
i. Substantive: law that defines duties, relationships, and rights of the parties external to the
litigation process
ii. Procedural: law that determines the means by which the substantive rights are determined
in court

II. History
a. Erie: Man is hit while walking close to RR tracks and sues the RR
i. Overturned Swift because... (even though neither party argued to overturn Swift)
1. Misinterpreted the Rules & Decisions Act (said ALL laws—not just some state law)
2. Inequitable—denied equal protection of the law and encouraged forum shopping
3. Swift was unconstitutional because it violated const. limits—the fed cts had NOT
been delegated the law-making power of state law (e.g. torts and contract claims)
ii. Held that in div. cases substantive = state law, procedural = fed law, and NO MORE FED
COMMON LAW!
iii. Outcome determinative test: Look to see if it “significantly affects the result of
litigation”—if it does, apply state law
iv. Balancing test
b. Hanna: D served according to FRCP 4- argues that state law serves differently, thus service
doesn’t
count
i. Outcome determinative in York MEANS those procedural rules whose effect is so vital
that to apply different state and fed rule would cause forum shopping
ii. If there is a valid and applicable (on point) FRCP, then it applies no matter what—
iii. Two ways to determine if procedural:
1. Is there an FRCP on point?
2. If not, then follow the relatively unguided Erie choice

III. Policies
a. Limit forum shopping (gamesmanship)
b. Promotes vertical uniformity
c. Equitable administration of the law
IV. Flowchart
a. Is there a valid FRCP on point, controlling, and in conflict with state law? If yes, use federal
law
i. Two Steps: Ask...
1. Is FRCP Constitutional- Rationally procedural
a. Can anyone think that it has something to do with procedure
2. Does it violate the Rules Enabling Act- FRCP canNOT abridge, modify, or enlarge a
substantive right
a. If it does violate it, the FRCP is unconstitutional and state law applies

b. Is it state substantive law? If yes, use state law


i. Procedural- Manner or means by which substantive rights are determined

Downloaded From OutlineDepot.com


ii. Substantive- Defines duties, relationships, or rights

PLEADING (COMPLAINT AND ANSWER)


I. Generally
a. Purposes
i. Putting parties on notice of claims and defenses of their opponents
ii. Stating facts each party believed it could prove
iii. Narrowing the number and scope of issues needing trial
iv. Providing a quick method for resolving meritless claims and defenses

II. Complaint
a. Types—FRCP 7(a) Lists 7 types of pleadings that are allowed (can be summed up in three
categories)
i. Complaint, Answer, and Reply
b. Requirements—FRCP 8(a)
i. Claim for relief must contain the following: (Three things)
1. “Short and plain statement of the ground for the courts jurisdiction”
a. This is for SMJ
2. Short and plain statement of the claim showing the pleader is entitled to relief
3. A demand for relief sought (typically a monetary amount)
a. The demand does NOT limit the claim—you can prove more or less at trial
b. You do NOT have to prove or put an exact amount—you can just put “an
amount to be proven at trial”

ii. Failure means the complaint can be dismissed by a 12(b)(6) motion “failure to state a
claim...”
1. Even if all allegations are true, it is still insufficient
2. Usually 1 opportunity to amend, then dismiss with prejudice

c. Sufficiency
i. Current Rule:
1. General Rule: “Showing” under FRCP 8(a)(2) requires that the facts are plausible
and legal conclusions are not entitled to presumption of truth
2. Twombly- Include enough facts that the claim is plausible, possible is not enough,
but it does NOT have to be probable
a. Possible < plausible < probable
b. Plausibility rule- Must have enough facts that there is a reasonable
expectation that discovery will show that you are entitled to relief
c. Labels and legal conclusions are NOT sufficient—they must be supported
by underlying facts, enough facts to show you’re entitled to relief
i. E.g. don’t say “they were negligent,” say “they were drunk and hit
me”

3. Iqbal- (5-4 decision) Confirmed that Twombly didn’t just apply to antirust
a. Two prong approach: “The tenet that a court must accept as true all
allegations contained in a complaint is inapplicable to legal conclusions”
i. Legal conclusions are NOT entitled to presumptions of truth
1. Court will strike down legal conclusions
2. What is left, after striking parts, is read in light of prong 2
ii. The claim must be plausible (Twombly plausibility requirement)
1. Look for “reasonable inference that the defendant is liable”
using “judicial experience and common sense”

d. Heightened Pleading—FRCP 9(b)


i. “Heightened pleading” is required for certain things under FRCP 9(b):
1. Fraud

Downloaded From OutlineDepot.com

2. Mistake
ii. “Peculiarity” in FRCP means “heightened pleading”
iii. CanNOT require “heightened pleading” through judicial interpretation—must be done by
congress
e. Pleading in the alternative: Inconsistent theories are OK to plead, but you can only recover
on one

III. Answer

a. Two options:
i. Answer the complaint
1. Time Limit: Must answer w/in 21 days, or 60 if D waive service, or 90 days if out of
the country
2. Answer must contain two things: FRCP 8(b)
a. Admit or deny any allegations asserted against you by the opposing party
i. General Denial: Deny everything
ii. Specific Denial: Deny individual allegations
iii. Qualified Denial: Rephrase part that is true and otherwise deny
iv. Admit
v. Lack of knowledge or info insufficient: To form a belief about the
truth of an allegation
vi. Failure to deny is treated as an admission (is admitted as true
forever)
vii. Once you admit, you cannot go back on the issue
b. Affirmative defenses: State in short and plain terms Party’s defense
i. FRCP 8(c) and 12(b)(6) list affirmative defenses
ii. If 12(b)(2-5) and 8(c) are NOT included in answer, they will be
waived
iii. There is a big argument that you must provide evidence of fact to
support all of your affirmative defenses that are listed—this would
follow Twiqbal, but the lower courts are split on this issue and since
Twiqbal relied on “showing” but “showing” doesn’t appear in 8(b)
defenses

ii. Bring a motion


1. FRCP 12(b)- Present Defenses
a. May bring the 12(b) motions—if you don’t bring any you waive 12(b)(2-5)
b. 12(b)(6-7)- either can be made any time before or after trial
c. 12(b)(1)—SMJ can be raised anytime
2. FRCP 12(e)- Motion for a more Definite Statement
a. Move for a more definite statement to which a responsive pleading is
allowed, but which is so vague/ambiguous that the party can’t reasonably
prepare an answer
b. Elements
i. State the defect AND details you want
c. If 12(e) is granted, the P has 10 days to fix the complaint or the court will
strike it
3. FRCP 12(f)- Motion to Strike (very rare)
a. Court may strike from a pleading an insufficient defense or any redundant,
immaterial, impertinent, or scandalous matter
i. May do it on its own
ii. Or on motion by a party either before responding to the pleading OR
within 21 days after being served with the pleading

IV. Amended or Supplemental Pleadings—FRCP 15

a. Amendments before trial- 15(a)

Downloaded From OutlineDepot.com

i. Matter of course amendments-15(a)(1) May amend your pleading as a matter of course


within:
1. 21 days after serving it, OR
2. If the pleading is one to which a responsive pleading is required, 21 days after
service of a responsive pleading or 21 days after service of a motion under any of
12(b, e, f)
3. The other party has 14 days after service of the amended pleading to respond
ii. Other Amendments-15(a)(2) In all other cases, a party may amend its pleading ONLY
with...
1. Opposing party’s written consent, OR
2. Court’s leave (when justice so requires)

b. Amendments during and after trial- 15(b)


i. Court should “freely permit an amendment when doing so will (1) aid in presenting the
merits and (2) the objective party fails to satisfy the court that the evidence would prejudice
that party’s action or defense on the merits

c. Relation back to amendments- 15(c) (Deals w/ Statute of Limitations and Amending)


i. An amendment to a pleading relates back to the date of the original pleading when:
1. The law that provides applicable SOL allows relation back,
a. Look at states SOL to see if you can
2. Amendment asserts a claim or defense that arose out of the conduct,
transactions, or occurrence set out—or attempted to be set out—in the original
pleading
a. This usually means if it comes out of the same set of facts, it is OK to amend
3. The amendment changes the party or the naming of the party against the claim is
asserted and that party knows or should have known that the action would have
been brought against it, but for a mistake concerning the proper party’s identity
a. Failure to not know who the person is does NOT count—it has to be a
mistake!
i. Lack of knowledge does NOT = mistake
V. Veracity in Pleadings/Sanctions—FRCP 11
a. Everything submitted to the court MUST have the following things: 11(a)
i. Signature by one attorney of record in the attorney’s name OR by the party if they are pro se
ii. Signer’s contact information, email, phone number, address, etc...
iii. Unsigned paper MUST be stricken by the court UNLESS it is promptly corrected by the
attorney after being called to the attorney’s or party’s attention

b. Representation in the court—11(b)


i. Submission to the court certifies that the paper certifies that to the best of the person’s
knowledge, information, and belief, formed after an inquiry reasonable under the
circumstances:
1. It is not for an improper purpose—such as to harass, cause unnecessary delay, or
needlessly increase the cost of litigation, AND
2. The legal claims are warranted by existing law or by a non-frivolous argument
for extending, modifying, or changing existing law, AND
3. Factual contentions have evidentiary support or are likely to have evidentiary
support after a reasonable opportunity for further investigation after discovery,
AND
4. The denials or factual contentions are warranted on the evidence or, if
specifically so identified, are reasonably based on belief or a lack of information

ii. NOTES:
1. Do NOT forget the first part that it was formed after a reasonable inquiry under the
circumstances

Downloaded From OutlineDepot.com


a. If you have one day to file b/c of the SOL, you can take your client’s word for
it
c. Sanctions—11(c)
i. If the court determines that 11(b) has been violated, the court may impose an appropriate
sanction on ANY attorney, law firm, or party that violated the rule—absent exceptional
circumstances, a law firm must be held jointly responsible for a violation committed by
its partner, associate, or employee
ii. Two ways sanctions may be imposed: 11(c)(2-3)
1. By Motion
a. Motion for sanction must be made separately from any other motion, AND
b. Must describe the violations of 11(b), AND
c. You send it FIRST to the opposing party and give them 21 days before you
file the motion sanction, to fix the problem
i. Safe harbor rule: Party has 21 days after motion to correct a motion
that was incorrect

d. If warranted, court may award to the prevailing party the reasonable


expenses, including attorney fees, incurred for the motion

iii. Nature of the Sanction—11(c)(4)


1. Must be limited to what suffices to deter repetition of the conduct or comparable
conduct by others similarly situated
2. May be non-monetary directives:
a. Striking, CLE requirements, Admonition, Censure, Reporting lawyer to the
bar

3. May be a monetary penalty


a. Fine payable to the court
b. Reasonable attorney fees and other expenses relating to the violation
payable to the movant
c. Monetary sanctions are NOT appropriate when:
i. The party is making 11(b) legal argument (frivolous, bad), OR
ii. Can NOT do it against the party—must be against their attorney

DISCOVERY
I. Generally: The quantity and time of the information sought is left almost entirely to the
parties with judicial
intervention only when there is a problem
II. Scope of Discovery: FRCP 26(b)
a. Parties may obtain discovery regarding any non-privileged matter that is relevant to any
party’s
claim or defense
i. Relevant Matter
1. Relevance- Need a “cogent nexus” between the information that you want and the
claim or defense that you are bringing
2. Must be for existing claims/defenses—can’t try to find new ones
3. Requesting party has burden to show relevance
ii. Privileged Matter
1. Once disclosed, it is no longer privileged
a. If you disclose part of communication on a subject, rest of communication on
the subject is waived

2. Privileges
a. Attorney/client—four elements
i. Confidential
ii. Communication
1. It is ONLY communication that is privileged

Downloaded From OutlineDepot.com


iii. Between attorney and client
iv. For the purpose of obtaining legal advice
v. NOTE: If there are other parties present, the privilege is destroyed
b. Work/product immunity—26(e)
i. All trial preparation by an attorney
1. Qualified immunity- Such as notes for interviews, courts,
etc...
a. CAN be discoverable IF a “substantial need”—not
available by other means

2. Absolute immunity- Thought, conclusions, opinions,


impressions

c. Doctor/patient
d. Priest/penitent
e. Spousal
3. Privileged Documents- 26(b)(5)
a. When a party withholds information otherwise discoverable by claiming
that it is privileged, the party must:
i. Expressly make claim that it is privileged, AND
ii. Describe the nature of the documents in such a manner that, w/o
revealing info itself privileged or protected, will enable the other
party to assess the privilege claim
1. Can’t just hide things—must disclose

III. Discovery Devices:


a. Initial Disclosure-FRCP 26(a) A party, w/o awaiting a discovery request, must provide to other
parties:
i. Name and contact information of each person likely to have discoverable information you
may use to support your claim or defense
ii. A copy/description of all documents, including tangible things, the party has in their control
to support its claims
iii. A computation of each category of damages
iv. Any insurance agreement under which an insurance business may be liable
b. Supplementing Disclosures and Responses- (FRCP 26(e)) A party who has made a disclosure
under 26(a), must supplement or correct its disclosure or response:
i. In a timely matter if the party learns the response/disclosure is incomplete or incorrect; OR
ii. As ordered by the court
iii. Generally: This rule means that if you get a new document you have to disclose anything
new and correct anything that you gave them that might not be correct anymore

c. Disclosure of Expert Witnesses—FRCP 26(a)(2)


i. Ordinary witnesses cannot express opinions, only experts where technical/specialized
knowledge will assist trier of fact to understand the evidence
ii. A party MUST disclose to the other parties the identity of any experts that may be used at
trial to present evidence—must disclose at least 90 days before trial
1. Note: If the witness is NOT disclosed, they can NOT be used at trial (see 37(c))
a. Two exceptions: Unless if substantially justified OR harmless

iii. Unless otherwise stipulated or ordered by the court, this disclosure MUST be accompanied
by a written report—prepared and signed by the witness; report must contain:
26(a)(2)(B)
1. Complete statement of all opinions the witness will express and basis/reasoning,
2. Data other information considered by the witness in forming them,
3. Any exhibits that will be used to summarize or support them,

Downloaded From OutlineDepot.com

4. The witness’s qualifications, including list of all publications authored in previous


10 yrs,
5. List of all other cases in which, during the previous 4 years, the witness testified as
an expert at trial or deposition,
6. A statement of the compensation to be paid for study and testimony in the case
iv. Categories of witnesses
1. Experts a party expects to use at trial
2. Experts retained or specifically employed, but NOT expected to be used at trial
3. Experts informally consulted in preparation for trial, but not retained
a. No discovery may be had of the names or views of experts in this category
4. Experts whose information was not acquired in preparation for trial
a. These are experts that are in fact witnesses and are accessible by either
party

v. Factors to determine if the expert is retained/specially employed (2) or informally


consulted (3)
1. Are they paid (specially employed)
a. Extent of time/depth of consultation
2. Decide not to use the witness = Informally consulted

d. Depositions—FRCP 30-32
i. Deposition is a sworn statement
1. Used for impeachment of witnesses and evidence for summary judgment
ii. Limitations:
1. Generally, each side has 10 depositions
a. Can be changed by agreement or court order
2. Time (1 day with a 7 hour maximum per day per deposition)
a. Can be altered by agreement
3. Anyone with relevant information can be deposed—they don’t need to be a party to
the issue
a. If you don’t know who will have the best information, the corporation MUST
name the person that does
b. Must provide the name of the organization, information you want, and the
corporation must find the person and present them
4. Cannot be used at trial except for impeachment OR if not available
5. Very expensive to do
e. Interrogatories—FRCP 33
i. Written question to opposing side
ii. Can ONLY be sent to parties (this is a BIG difference from a deposition)
iii. Written by an attorney and are carefully crafted
iv. Used to get basic information and answers to research
v. Helpful to get BEFORE a deposition, so you know what to ask in deposition
f. Document Requests—FRCP 34
i. Permits a party to require another party to produce for inspection, copying, or testing all
relevant documents and tangible things
ii. Unlimited—you get as many as you want
iii. Get these BEFORE your deposition so that you can get the ?s you want to ask in your
deposition

g. Medical Examinations—FRCP 35
i. Must show good cause and a mental/physical condition must be in controversy—must be
specific about what you cover or ask for
ii. Limited to parties to the suit (or extremely closely tied to the suit)
h. Request for Admission—FRCP 36

Downloaded From OutlineDepot.com


i. Used to determine what issues are and are not in dispute
1. Used to get the other side to admit facts that are not in dispute
ii. There are sanctions if you don’t admit facts that are not in dispute
i. Conference with Parties—FRCP 26(f)
i. Parties must confer as soon as practicable—and in any event at least 21 days before a
scheduling conference is to be held or a scheduling order is due under 16(b)
ii. Discuss settlement, initial disclosures, and set up a proposed discovery plan w/ opposing
attorney
iii. Within 14 days after 26(f) conference, must make initial disclosures (R26(a)), leaving 1
week until 16(a) scheduling conference
j. Scheduling Conference and Order—FRCP 16
i. Conference covers all aspects of the trial and sets deadlines for everything
ii. Must live by the scheduling order—it is binding absent showing really good cause
iii. Scheduling order is issued as soon as practicable, but it is within the earlier of (1) 120 days
after any defendant has been served with the complaint OR (2) 90 days after any D has
appeared
iv. Whole process results in two orders:
1. Scheduling Conference—16(b)—Scheduling Order
a. Sets all deadlines, close of dispositive motions, can only be modified with
“good cause” and the judges consent
2. Final Pretrial Conference—16(e)—Pretrial Order
a. Held right before trial
b. Contains
i. Each party’s assertion of facts/legal issues
ii. Every witness to be called
iii. Every piece of evidence that will be submitted
iv. Every piece of relief that you want
c. Will NOT be modified unless there will be “manifest injustice”
d. Supersedes all pleadings and governing actions

k. Timeline Summary
i. 26(f) conference with opposing counsel
ii. Scheduling conference and order (happens w/14 days of conference with opposing counsel)
1. Tells when the date for close of discovery
2. Tells when the date for close of dispositive motions
3. Sets a trial date
iii. 30 days before trial—pretrial disclosures
1. Names of witnesses to be called, evidence to be used, and
depositions/interrogatories to be used at trial must be disclosed
a. Experts are disclosed 90 days before trial

iv. 16 days before trial—file objections to opposing counsel’s arguments/pretrial disclosures


1. You object before trial even happens
v. Final pretrial conference—held as close to the trial as possible
1. Receive the pretrial motion/order
a. This motion supersedes everything (even previously dictated date schedule)

l. Signing Disclosures and Discovery Requests/Responses—FRCP 26(g)


i. Similar to Rule 11—all documents MUST be signed by at least one attorney and must give
their contact information (email address, phone number, address)
ii. By signing, the attorney certifies that they formed their belief after a reasonable inquiry
AND it is to the best of their knowledge or information; they certify that:
1. It is complete and correct, AND
2. Consistent with the rules and warranted by existing law, AND

Downloaded From OutlineDepot.com

3. Not interposed for any improper purpose such as to harass, cause unnecessary
delay, or needlessly increase the cost of litigation, AND
4. Neither unreasonable nor unduly burdensome or expensive, considering the
needs of the case, prior discovery in the case, the amount in controversy, and the
importance of the issues at stake in the action
a. THIS section (4), is different than Rule 11
IV. Discovery Sanctions—FRCP 37
a. A party may move for an order to compel discovery, BUT the movant MUST certify that they
conferred or attempted to confer with the other party or the party failing to make
disclosure/discovery BEFORE involving court action/assistance
b. Steps to Discovery Problems:
i. Confer- with opposing counsel and ask them for the desired material
ii. Compel- Go to the court and move for an order to compel disclosure or discovery
iii. Court rules on the motion
1. If they rule to compel, the court MUST require the losing counsel to pay reasonable
expenses, including attorney fees:
a. Two exceptions to this:
i. The movant filed the motion BEFORE attempting in good faith to
obtain the disclosure/discovery without court action
ii. The opposing party’s nondisclosure, response, or objection was
substantially justified

2. If they do NOT rule to compel, the court MUST require the moving party to pay
reasonable expenses of the other party in answering the motion
a. Only one exception here: No penalty if motion was substantially justified
iv. Losing party must give over the info. (and pay the fees—unless it meets one of the
exceptions)
1. If you don’t comply, you will be found in contempt of court
a. Attorneys will only use this option when the attorney really feels that giving
up will be too damaging to their client and they want a quick chance at an
appeal
b. Only happens in rare and extreme circumstances
2. Court may also do any of the following: 37(b)(2)
a. Direct the matters wanted to be taken as fact,
b. Prohibit disobedient party from supporting or opposing designated claims
or defenses,
c. Striking pleadings in whole or in part,
d. Staying further proceedings until the order is obeyed,
e. Dismissing the action or proceeding in whole or in part, or
f. Rendering default judgment
3. Instead of or in addition to what is just mentioned, the court must order payment of
fees

c. Failure to Disclose or Supplement—37(c)


i. If a party fails to provide information or identify a witness required by Rule 26(a), the
party is NOT allowed to use that information or witness to supply evidence on a motion, at a
hearing, or at a trial, UNLESS the failure was...
1. Two Exceptions:
a. Substantially justified, OR
b. Harmless
RIGHT TO A JURY TRIAL
I. In 1938, FRCP merged the legal and equitable courts

Downloaded From OutlineDepot.com

II. Amendments
a. 7th Amendment applies to civil cases (does not apply to states—thus in a civil court case,
there is no
federal constitutional right to a jury, though a state constitution may provide for one)
i. “Suits at common law” refers to suits in the courts of law as opposed to the courts of equity
ii. It provides that “the right of trial by jury shall be preserved”

III. Test for Whether Jury Trial is Required


a. Preserve = Did you have a right to a jury trial in 1791?
b. Two types of claims to decide:
i. Legal- Jury
ii. Equitable Claims- Bench Trial
c. Two part test to determine which kind of claim it is:
i. 1791 Analog (Legal History Prong)
1. See whether the instant case’s cause of action was in equity or legal
2. If it was NOT a claim in 1791, find an analogous cause of action in 1791 and find out
whether or not it relates or is analogous to the instant case
ii. Type of Remedy Involved (trumps part 1 if they conflict)
1. Look to see if it is a legal or equitable remedy
2. Monetary relief, OR

IV. Mixed Claims and Mergers


a. FRCP merged equitable and legal court & permits parties to join legal and equitable claims in
a
single suit
b. You will try the legal one first (with a jury), and the equitable claims second
i. You do this b/c if you try equitable ones first, the facts will be assumed true in the legal case
(which is tried second) and the jury wouldn’t be able to rule on the facts (contrary to their
right)
JURY SELECTION
I. Peremptory Challenges
a. Number is fixed by statute:
i. Federal civil cases- 3
b. Reasons for peremptory challenges
i. Allows for D to equalize their position
ii. Legitimizes verdicts since both sides pick the jury
iii. Allows lawyers to strike people w/o spending time and effort
c. Traditionally, attorneys could strike anyone for any reason
d. Can NOT strike for gender or race reasons

DISPOSITIONS OF A CASE
I. Summary Judgment—FRCP 56
a. Generally
i. Use before going to trial
ii. Summary Judgment is used when there are just issues of law
iii. All evidence considered is in written form
b. Timing of motions—56(c)
i. A party may move for summary judgment any time until 30 days after the close of all
discovery,
ii. A party opposing the motion must file a response within 21 days after the motion is served
or a responsive pleading is due, whichever is later; and
iii. The movant may file a reply within 14 days after the response is served
c. Appropriate when “there is no genuine issue as to any material fact and when the movant is
entitled to judgment as a matter of law”—56(c)(2)

Downloaded From OutlineDepot.com

d. If opposing party moves for SJ, but there hasn’t been time for discovery yet to disprove their
assertions, the court will grant time for discovery—56(f)
e. SJ v. 12(b)
i. SJ is different than 12(b) b/c in 12(b) the court relies solely on pleadings to determine the
facts
ii. In contrast, in ruling on SJ, the court looks beyond the pleadings and considers material
such as affidavits or other sworn statements such as depositions or interrogatory answers
iii. SJ is often called “piercing the pleadings”—you look beyond the pleading to see if there is
anything there to support your claim or facts that you need

f. Standard
i. View the evidence in the light most favorable to the nonmoving party (resolving inferences
in favor of the nonmoving party), and movant must prove that no reasonable jury could find
(under the appropriate standard: preponderance of evidence < clear and convincing
evidence < beyond a reasonable doubt) for the nonmoving party
1. The court can NOT weigh the evidence or make judgments
a. Ex: If you have 1 witness and they have 15, you are OK, if you are the only
witness on your side and the other side has 15, SJ is still NOT appropriate
b. Circumstantial evidence is good enough to defeat SJ—even if other side has
direct, contradictory evidence

2. Materiality
a. Not all factual disputes are material—must be about a critical substantive
fact
g. Burden Standards and Responding to SJ
i. Preponderance of evidence (51%+), clear and convincing evidence (highly probable or
reasonably certain), and beyond a reasonable doubt
ii. Burden of Production
1. The burden the moving party bears when they make the motion for SJ
iii. Burden of Persuasion
1. This lies with the person that has the burden of proof (this could be on the D or P)
iv. Celotex (Wife claimed husband died from Asbestos from D’s products)
h. Bench Trial
i. This does NOT change the standard for SJ
1. If you did change the standard and allowed the judge in a bench trial to just take
care of it in SJ, it would become a paper trial where the judge merely rules on the
affidavits and not on all the evidence
II. Judgment as a Matter of Law (JMOL) (FRCP 50)
a. JMOL = Directed verdict
i. Occurs BEFORE the jury comes back with their decision
b. Process
i. A motion is required if a party wants a JMOL from the court
ii. Motion MUST be made AFTER all evidence from both parties has been presented
1. Ex: P can’t move for JMOL after they just present their evidence

c. Differences from SJ
i. SJ is pretrial, JMOL is after the trial has began
ii. SJ is based on discovery, but JMOL is based on evidence at trial in court
d. Summary
i. 12(b)(6) motions- based on pleadings
ii. SJ- based on discovery
iii. JMOL/RJMOL- based on testimony/evidence at trial
iv. All of these take away things from the jury

III. Renewed JMOL (FRCP 50)

Downloaded From OutlineDepot.com

a. RJMOL = Judgment notwithstanding the verdict


i. Occurs AFTER the jury has given their verdict
b. Process
i. MUST first move for RJMOL BEFORE the jury comes back with their verdict
ii. Then, after verdict is entered, you MUST renew your RJMOL motion
1. This must be done “no later than 28 days after the entry of judgment”
a. This may be filed jointly or alternatively with a motion for new trial
b. No previous motion is needed for a new trial, just JMOL

iii. If the court grants RJMOL, it MUST conditionally rule on the new trial in case the ruling is
reversed or vacated
iv. On appeal, if the appellate court reverses/vacated the renewed RJMOL, then it has 3
options:
1. Reinstate the verdict
2. Grant a new trial
3. Remand to trial court determine if there should be a new trial

IV. Motion for New Trial—FRCP 59


a. Granting a new trial doesn’t take it away from all juries, just the one that decided it b/c they
weren’t
reasonable
b. Process:
i. Motion for new trial must be filed no later than 28 days after the entry of judgment—
59(b)
1. If motion is based on affidavits, they must be filed with the motion, and the opposing
party has 14 days to respond—59(c)

ii. The court can on its own, order a new trial for any reason that would justify granting one no
later than 28 days after the entry of judgment—59(d)

c. Reasons for a new trial:


i. Insufficiency of evidence
1. New trial is given if the prior verdict was contrary to a “great weight” of evidence
2. The judge weighs the evidence and if it is REALLY one sided, he can grant a new
trial
3. Look to see if there is a insufficient evidence as to a material fact
ii. Prejudicial conduct
1. Judge and attorney
a. Ex: Ex parte contact with jurors, alluding to matter not relevant or not
supported by admissible evidence, or lawyer expresses personal belief

iii. Juror Misconduct


iv. Excessive Damages
1. Must “shock” the conscience of the court
2. Remittur- Original amount was too high, offer a lower amount or a new trial
v. Newly discovered evidence
1. Has to be evidence that could NOT have been discovered with due diligence before
the judgment was rendered
2. Has to likely have changed the outcome of the case
3. It can NOT be information that you had all along
vi. Jury Instructions
1. Before the close of evidence, the attorney must submit proposed instructions to the
judge
2. Then, you have an instruction conference with the judge
a. Review instructions judge plans to give
b. Make objections to any instructions on the record
c. Judge decides which instructions will be given

Downloaded From OutlineDepot.com


i. If you fail to object, or object w/o a reason (why they are incorrect
statements of law), then appeal on the issue is waived
3. It is REALLY good to get this b/c it is de novo review on appeal
4. Errors in jury instructions are common basis for appeal and reversal
5. You MUST object to jury instruction in the trial court before you can appeal the
instructions on appeal

APPEALS
I. There is NO constitutional right to an appeal
a. §1291 gives appellate court jurisdiction over all final judgments of the district courts
II. Final Judgment:
a. A judgment that “ends litigation on the merits and leaves nothing for the court to do but
execute the judgment”
b. Interlocutory decision: A decision that comes before the end of the case and thus, is NOT
immediately appealable under §1291

III. Process:
a. After final judgment is entered, the party has 30 days to file an appeal or 60 days if the US is
a
party
b. The judgment is effective immediately after the judgment is entered
i. This means that you have to pay the judgment even while the appeal is going on

IV. Interlocutory Appeals: Exceptions to the final judgment rule


a. §1291(a)—Injunctions
i. Anything about granting, continuing, modifying, refusing or dissolving injunctions, or
refusing to dissolve or modify injunctions, you can get an interlocutory appeal on it
1. Meaning, anything dealing with an injunction means that you can get an
interlocutory appeal as a right
2. The reason we have this for injunctions is that irreparable harm is being done and
you can’t wait for the case to be over to appeal because the harm would continue
throughout the original trial until the final judgment is rendered

b. FRCP 54(b)—Multiple claims/parties


i. When an action presents more than one claim for relief or when multiple parties are
involved, the court may direct entry of final judgment as to one or more, but fewer than all,
claims or parties if the court expressly determines there is no just reason for delay
1. This means that you don’t have to wait around for the other parties in the suit to
finish, you can appeal your suit/portion that was disposed of early on
ii. Process:
1. District court MUST enter a final judgment as to the claim, AND
2. Expressly say there is “no just reason for delay”
a. Must expressly state that—its similar to certification

iii. After this occurs, you still have a final judgment and thus have a right to appeal under
§1291

c. §1292(b)—Question of law that is key to the whole case


i. Must have the following:
1. “An order”
2. “Involving a controlling question”
3. “Of law”
a. It can only be a legal question—not a question of fact or an application of a
fact to law

4. “As to which there is a substantial ground for difference of opinion”


a. Can’t be too one sided
b. There needs to be a split among the authorities

Downloaded From OutlineDepot.com

5. “That an immediate appeal from the order may materially advance the ultimate
termination of the litigation”
a. This means that the case can’t go forward if they don’t have an answer to
this particular question/element

ii. District court MUST certify that all five of these elements are true—they have to give you
permission to appeal by certifying this
iii. Then, the appellate court must agree to the certification of the five elements
1. Appellate court has complete discretion on whether or not to accept it
iv. Summary: There must be a question of law that is key to the whole case that is split in
authority and the court does NOT know what to do in the situation/case

FRCP 60-EXTRAORDINARY RELIEF FROM JUDGMENT


I. Counsel returns back to the trial court that issued the final judgment and asks that the final
judgment be
undone
a. This is NOT an appeal—you are going back to the court and asking them to change what they
said
b. Rarely happens—must be an “extraordinary situation”
c. You have a “reasonable time” to make a 60(b) motion—60(c)
II. Reasons for Extraordinary Relief from Judgment—60(b)
a. Mistake, inadvertence, surprise, or excusable neglect
i. Mistake has to be one of fact—not a law—otherwise you would have appealed it to the
Appellate Court
ii. Excusable neglect does NOT include failure to read or understand the law or rules
b. Newly discovered evidence that, with reasonable diligence, could NOT have been discovered
in
time to move for a new trial under FRCP 59(b);
i. Evidence is likely to change the outcome of the case
c. Fraud, misrepresentation, or misconduct by an opposing party
d. The judgment has been satisfied, released, or discharged; it is based on an earlier judgment
that
has been reversed or vacated; or applying it prospectively is no longer equitable; OR
i. This means preclusion

PRECLUSION
I. Claim Preclusion (AKA Res Judicata)
a. Definition: Precludes all claims that could have been brought in the first case
b. Elements: (Three + Result)
i. Final judgment on the merits
1. 41(b)—Pretty much everything that is not “lack of jurisdiction, improper venue,
or failure to join a party under Rule 19” is ON THE MERITS
2. “On the merits” means “where there was an opportunity to prosecute on the merits”
ii. Same parties or their privies (nonparties)
1. Nonparty preclusion may be justified based on a variety of preexisting “substantial
legal relationships” between the party to be bound and a party of the judgment
i. Ex: If you have a property dispute with neighbor, and it is settled,
someone that buys your house cannot relitigate the issue

2. A nonparty may be bound by a judgment b/c she was “adequately represented by


someone with the same interests who was a party to the suit’

i. Ex: Class actions, fiduciaries, parents

iii. Same claim—3 Different Theories


1. Primary Rights: We define a claim by deciding if it is the same right
a. Ex: Right to secure property is different than the right to secure person
2. Single Wrongful Act: Look at the wrongful act that caused everything

Downloaded From OutlineDepot.com


a. Ex: If someone slanders you and punches you, that is two different acts; but,
when he punched you he ruined your shirt, that would be a single wrongful
act even though you had property and personal damages

3. Same Transaction or Occurrence (T/O): This is the federal rule (federal common
law) and the majority view- Focuses on whether the facts are connected in time or
manner AND common nucleus of operative fact

iv. Result: It precludes “all matter that were or might have been litigated”

II. Issue Preclusion (Aka Collateral Estoppel)


a. Definition: Not allowed to re-litigate an issue that has been decided by a previous lawsuit
b. Elements: (Three)
i. Final judgment on the merits
1. Default judgment and 12(b)(6) will not bring up issue preclusion
ii. Precludes a party or privy from first case
1. This is different than “claim” preclusion analysis—may be mutuality, DNMCE,
ONMCE

iii. From re-litigating the same issue—two part test:


1. Actually litigated and decided, AND
2. Necessary and essential to the judgment

c. Mutuality- Required to have the same plaintiff and defendant from prior action to use
preclusion
d. Defensive Non-Mutual Collateral Estoppel (DNMCE)
i. Rule: Used by D, P sues D1 and loses on an issue, then sues D2 on the same issue= D2 can
use the decision on the 1st case to preclude P from re-litigating the issue
ii. D1 (wins)

P1 (loses)
D2

e. Offensive Non-Mutual Collateral Estoppel (ONMCE)


i. Rule: Used by P, D is precluded from using a defense/issue decided in the 1st case
ii. P1 D1 (loses)
(win)
P2
iii. District Court has discretion to decide if it is allowed
1. Elements of discretion:
a. Ease of joining
i. If P could have easily joined the 1st trial then he may be precluded
from using ONMCE

b. Judgment is inconsistent with previous decisions


c. Different procedural opportunities
i. Ex: You had no discovery in the first action
1. Jury trial is NOT enough of a procedure for preclusion
d. Incentive to litigate or foreseeable
i. If in the first judgment you could not foresee that there were going
to be future cases and the amount was small, you may not zealously
defend your position and lose—thus opening up a floodgate.

JOINDER AND SUPPLEMENTAL JURISDICTION


I. Generally
a. Step 1: Ask if there is a joinder rule that permits assertion of the claim
i. If no, you cannot continue
ii. If yes, continue to step two
b. Step 2: Ask does the court have jurisdiction (PJ, SMJ, venue, etc...) to hear the claim?

Downloaded From OutlineDepot.com

i. Must satisfy BOTH steps

II. FRCP 18- Joinder of Claims


a. Rule: Allows P to join/add as many claims as they want to their claim against the opposing
party,
but they must have an existing claim (or counterclaim) to use this rule
b. Covers claims, counterclaims, cross-claims, or 3rd party claims
c. Claims do NOT have to be related
d. Court MUST have jurisdiction to hear all of the claims asserted (need independent OR supp.
jurisdiction)

III. FRCP 14- Impleader of 3rd Parties


a. When a Defendant May Bring in a 3rd Party-14(a)
i. Rule: “D may serve a summons and complaint on a nonparty who is or may be liable to it
for all or part of the claim against it”
1. D can bring in a TPD even if it occurs before the TPD is liable to D—14(a) says “who
is or may be liable”

ii. Timing: D must obtain the courts leave, by making a motion, if it files the 3rd party
complaint more than 10 days after serving its original answer

b. 3
rd Party D’s Claims and Defenses-14(a)(3)
i. TPD must assert their 12(b) defenses in answer
ii. May assert against P any claim arising out of the same T/O that is the subject matter of P’s
complaint
1. However, make sure there is sufficient SMJ or SJ for the claim to be supported

c. When a Plaintiff may Bring in a 3rd Party-14(b)


i. Rule: “When a claim is asserted against a plaintiff, the plaintiff may bring in a 3rd party if
this rule would allow a defendant to do so”

IV. FRCP 20- Permissive Joinder of Parties


a. Plaintiffs: 20(a)(1)
i. Rule: P may join in one action as plaintiffs if:
1. The claim arises out of CNOF, AND
2. Any question of law or fact common to all P’s will arise in the action

b. Defendants: 20(a)(2)
i. Rule: P may join in one action as defendants if (same rule as plaintiffs)
c. Court’s Discretion: 20(b)
i. The court may issue orders—including an order for separate trials—to protect against
embarrassment, delay, expense, or other prejudice that arises from including a person
against whom the party asserts no claim and who asserts no claim against the party

d. Supplemental Jurisdiction
i. If federal question, allowed under 1367a
ii. If jurisdiction is based on diversity, no SJ if...
1. If this would destroy diversity OR
2. Claim doesn’t reach jurisdictional amount

V. FRCP 13(a) & (b)- Counterclaims


a. Compulsory- 13(a)
i. Rule: A pleading must state as a counterclaim any claim that the pleader has against an
opposing party if the claim:
1. Arises out of same T/O that is subject matter of the opposing party’s claim, AND
2. Does NOT require adding another party over whom the court cannot acquire
jurisdiction

ii. D may include a new plaintiff if CNOF and STO standards are met
iii. Discretion: Because compulsory counterclaim is an equitable doctrine, the court has
discretion and may let parties out of it if...
1. The plaintiff was pro se and didn’t have legal assistance

Downloaded From OutlineDepot.com

2. There is a settlement (rarely happens, but the court will state this isn’t a final
judgment and perhaps let you out of it)

iv. NOTE: Use it or lose it—if default judgment is entered against you, you lose that case AND
any compulsory counterclaims that you would have had in that case

b. Permissive- 13(b)
i. Rule: A pleading may state as a counterclaim against an opposing party any claim that is not
compulsory

c. Supplemental Jurisdiction for Compulsory Counterclaim


i. Rule: ALWAYS will have CNOF
ii. Federal Question: If you have a compulsory counterclaim on a case that is there b/c of a
federal question, you will ALWAYS have SJ because it arises out of CNOF
1. Just need to meet CNOF
a. Ex: Even if the compulsory counterclaim is a state claim, SJ will be given b/c
it has a CNOF with the jurisdiction conferring claim

iii. Diversity: Will ALWAYS have CNOF for diversity—even if jurisdiction amount on claim is
too small

d. Supplemental Jurisdiction for Permissive Counterclaim


i. Rule: Permissive counterclaims will NEVER have supplemental jurisdiction
1. They will always need independent jurisdiction

VI. FRCP 13(g)- Cross claims


a. Rule: A cross claim is permitted if...
i. The claims arises out of same T/O, OR
ii. The claim relates to any property that is subject matter of the original action
b. Note: CanNOT have a cross claim on something that is unrelated (this is different than a
counterclaim b/c with a counterclaim you can have it on something that is unrelated—called
permissive counterclaims)
i. Must have CNOF- this is odd b/c it is NOT compulsory to cross claim, but it is
w/counterclaims
VII. FRCP 19- Required Joinder of Parties

a. Process: 2 Steps
i. Step 1: Determine if the party is a “required party” (aka “necessary”)
1. Three ways a party can be “required”:
a. The court cannot provide complete relief without them, OR
b. It will impair or impeded absentee’s ability to protect their practical
interest, OR
c. It would subject the existing party to risk of double, multiple or inconsistent
obligations (Ex: Can’t put full amount of stock in the name of 2 people)

ii. Step 2: Determine if joining parties is “feasible”


1. Whether absentee is subject to PJ, would it invoke federal question and diversity

b. If party is NOT feasible


i. Rule: If joining party is not feasible, must determine if party is “indispensible”
1. Factors to determine if “indispensible”
a. Extent of prejudice to outsiders or existing parties
b. Extent to which this can be fixed through other means
c. Whether a judgment rendered in the persons abuses would be adequate
i. Adequacy: Determine if there is a public interest as doing it as a
whole, and if it will be complete, consistent, and efficient
d. Whether there is an alternate forum for this dispute where everyone can
be parties and they can be all be sued
i. Ex: All could be sued in state court
2. If indispensible, dismiss the case—if not, proceed

Downloaded From OutlineDepot.com

c. Supplemental Jurisdiction
i. Federal Question

Rule: Will have SJ because if they are required there is CNOF-which is enough for
supp
ii. Diversity
1. If brought under diversity and they are a D, they are NOT allowed under §1367(b)
OR
2. If brought under diversity and they are a P, they are NOT allowed under §1367(b)
3. Rule: No supp. jurisdiction for Rule 19 parties
a. You will always need independent SMJ for the party to be proper

VIII. FRCP 24- Intervention

a. Rule: On a timely motion, the court must permit anyone to intervene who:
i. Has an interest relating to the property or transaction that is the subject of the action,
ii. So situated that disposing of the action may as a practical matter impair or impede the
movant’s ability to protect its interest, or
iii. Existing party will adequately represent you
b. Supplemental Jurisdiction
i. Federal Question
1. Rule: Will have SJ because there will always be CNOF-which is enough for supp
jurisdiction
ii. Diversity:
1. Rule: No supp. jurisdiction allowed—will always need independent SMJ

IX. Jurisdiction
a. Independent: Always check for independent basis for SMJ (federal question and diversity)
b. Supplemental- §1367
i. Federal Question (Check (a) and (c) only)
1. Step 1: Is there CNOF with jurisdictional conferring claim? (a)
a. Same T/O = Common Nucleus of Operative Fact
2. Step 2: Does the case meet any of the discretionary factors to decline supp.
jurisdiction? (c)
a. Claim raises a novel or complex issue of state law
b. Claim substantially predominates over the claim or claims over which the
District Court has original jurisdiction,
c. District Court has dismissed all claims over which it has original jurisdiction,
or
d. Exceptional circumstances
i. Ex: Jury Confusion
ii. Diversity: (Check (a), (b), and (c))
1. Step 1: Is there CNOF with jurisdictional conferring claim? (a)
2. Step 2: Supplemental jurisdiction is NOT OK when...
a. Claims by plaintiff’s against persons made parties under Rule 14, 19, 20, or
24
b. Claims by persons proposed to be joined as plaintiffs under Rule 19
c. Claims by persons seeking to intervene as such plaintiffs under Rule 24
3. Step 3: Discretionary factors (see step 2 under federal question)

CPL-102 • Professor Ressler • BLS Fall 2016

I. SUBJECT MATTER JURISDICTION


U.S. Constitution, Article III, §1, Judicial Power, Tenure and Compensation.
The judicial Power of the United States, shall be vested in one supreme Court, and in such
inferior Courts as the Congress may from time to time ordain and establish.
The Judges, both of the supreme and inferior Courts, shall hold their Offices during good
Behavior, and shall, at stated Times, receive for their Services, a
Compensation, which shall not be diminished during their Continuance in Office.
U.S. Constitution, Article III, §2, Clause 1. Jurisdiction of Courts.
The judicial Power shall extend to all Cases, in Law and Equity, arising under this Constitution,
the Laws of the United States, and Treaties made, or which shall be
made, under their Authority;--to all Cases affecting Ambassadors, other public Ministers and
Consuls;--to all Cases of admiralty and maritime Jurisdiction;--to
Controversies to which the United States shall be a Party;--to Controversies between two or
more States;--between a State and Citizens of another State;--between
Citizens of different States;--between Citizens of the same State claiming Lands under Grants of
different States, and between a State, or the Citizens thereof, and
foreign States, Citizens or Subjects.
A. Federal Question Jurisdiction
i. U.S. Constitution Article III, §2 [see above]
ii. 28 U.S.C. §1331

§1331. Federal Question.


The district courts shall have original jurisdiction of all civil actions arising under the
Constitution, laws, or treaties of the United States.
iii. 28 U.S.C. §1257

a) [Osborne] If there is a federal ingredient, the case arises under the constitution »
very broad, old test that used to be applied to determine if case is "arising under"

§1257. State courts; certiorari


(a) Final judgments or decrees rendered by the highest court of a State in which a decision
could be had, may be reviewed by the Supreme Court by writ of certiorari
where the validity of a treaty or statute of the United States is drawn in question or where the
validity of a statute of any State is drawn in question on the ground of its
being repugnant to the Constitution, treaties, or laws of the United States, or where any title,
right, privilege, or immunity is specially set up or claimed under the
Constitution or the treaties or statutes of, or any commission held or authority exercised under,
the United States.
(b) For the purposes of this section, the term “highest court of a State” includes the District of
Columbia Court of Appeals.
iv. What does "Arising under" language mean [as used in §1331]?
v. Well-Pleaded Complaint Rule
Motley » FQ jurisdiction cannot be invoked unless an issue of federal law appears on
the face of a properly pleaded complaint
Federal issue cannot be a defense that plaintiff anticipates [like in Motley]

vi. Holmes Creation Test

a) Motley [Old Rule]: A case arises under the law that creates the cause of action
b) Smith [New Rule » Expands Holmes Test]: Since a federal issue (Federal Farm
Loan Act) was embedded in the state law claim and it was essential to the resolution,
FQ jurisdiction exists even though it fails the Holmes test.
vii. Substantial Federal Issue Approach » Modern "Arising Under" Rule
Grable v. Darue » State-law claims can be heard in federal court if the claim raises a
substantial federal issue that is actually disputed.

B. Diversity Jurisdiction
i. U.S. Constitution Article III, §2 [see above]
ii. 28 U.S.C. §1332
§1332. Diversity of citizenship; amount in controversy; costs
(a) The district courts shall have original jurisdiction of all civil actions where the matter in
controversy exceeds the sum or value of $75,000, exclusive of interest and
costs, and is between--
(1) citizens of different States;

Downloaded From OutlineDepot.com


(2) citizens of a State and citizens or subjects of a foreign state, except that the district courts
shall not have original jurisdiction under this subsection of an action
between citizens of a State and citizens or subjects of a foreign state who are lawfully admitted
for permanent residence in the United States and are domiciled in the
same State;
(3) citizens of different States and in which citizens or subjects of a foreign state are additional
parties; and
(4) a foreign state, defined in section 1603(a) of this title, as plaintiff and citizens of a State or of
different States.
(b) Except when express provision therefor is otherwise made in a statute of the United States,
where the plaintiff who files the case originally in the Federal courts is
finally adjudged to be entitled to recover less than the sum or value of $75,000, computed
without regard to any setoff or counterclaim to which the defendant may be
adjudged to be entitled, and exclusive of interest and costs, the district court may deny costs to
the plaintiff and, in addition, may impose costs on the plaintiff.
(c) For the purposes of this section and section 1441 of this title--
(1) a corporation shall be deemed to be a citizen of every State and foreign state by which it has
been incorporated and of the State or foreign state where it has its
principal place of business, except that in any direct action against the insurer of a policy or
contract of liability insurance, whether incorporated or unincorporated, to
which action the insured is not joined as a party-defendant, such insurer shall be deemed a
citizen of--
(A) every State and foreign state of which the insured is a citizen;
(B) every State and foreign state by which the insurer has been incorporated; and
(C) the State or foreign state where the insurer has its principal place of business; and
(2) the legal representative of the estate of a decedent shall be deemed to be a citizen only of
the same State as the decedent, and the legal representative of an infant or
incompetent shall be deemed to be a citizen only of the same State as the infant or
incompetent.
(d)(1) In this subsection--
(A) the term “class” means all of the class members in a class action;
(B) the term “class action” means any civil action filed under rule 23 of the Federal Rules of Civil
Procedure or similar State statute or rule of judicial procedure
authorizing an action to be brought by 1 or more representative persons as a class action;
(C) the term “class certification order” means an order issued by a court approving the
treatment of some or all aspects of a civil action as a class action; and
(D) the term “class members” means the persons (named or unnamed) who fall within the
definition of the proposed or certified class in a class action.
(2) The district courts shall have original jurisdiction of any civil action in which the matter in
controversy exceeds the sum or value of $5,000,000, exclusive of interest
and costs, and is a class action in which--
(A) any member of a class of plaintiffs is a citizen of a State different from any defendant;
(B) any member of a class of plaintiffs is a foreign state or a citizen or subject of a foreign state
and any defendant is a citizen of a State; or
(C) any member of a class of plaintiffs is a citizen of a State and any defendant is a foreign state
or a citizen or subject of a foreign state.
(3) A district court may, in the interests of justice and looking at the totality of the
circumstances, decline to exercise jurisdiction under paragraph (2) over a class action
in which greater than one-third but less than two-thirds of the members of all proposed
plaintiff classes in the aggregate and the primary defendants are citizens of the
State in which the action was originally filed based on consideration of--
(A) whether the claims asserted involve matters of national or interstate interest;
(B) whether the claims asserted will be governed by laws of the State in which the action was
originally filed or by the laws of other States;
(C) whether the class action has been pleaded in a manner that seeks to avoid Federal
jurisdiction;
(D) whether the action was brought in a forum with a distinct nexus with the class members,
the alleged harm, or the defendants;
(E) whether the number of citizens of the State in which the action was originally filed in all
proposed plaintiff classes in the aggregate is substantially larger than the
number of citizens from any other State, and the citizenship of the other members of the
proposed class is dispersed among a substantial number of States; and
(F) whether, during the 3-year period preceding the filing of that class action, 1 or more other
class actions asserting the same or similar claims on behalf of the same or
other persons have been filed.
(4) A district court shall decline to exercise jurisdiction under paragraph (2)--
(A)(i) over a class action in which--
(I) greater than two-thirds of the members of all proposed plaintiff classes in the aggregate are
citizens of the State in which the action was originally filed;
(II) at least 1 defendant is a defendant--
(aa) from whom significant relief is sought by members of the plaintiff class;
(bb) whose alleged conduct forms a significant basis for the claims asserted by the proposed
plaintiff class; and
(cc) who is a citizen of the State in which the action was originally filed; and
(III) principal injuries resulting from the alleged conduct or any related conduct of each
defendant were incurred in the State in which the action was originally filed;
and
(ii) during the 3-year period preceding the filing of that class action, no other class action has
been filed asserting the same or similar factual allegations against any of
the defendants on behalf of the same or other persons; or
(B) two-thirds or more of the members of all proposed plaintiff classes in the aggregate, and
the primary defendants, are citizens of the State in which the action was
originally filed.
(5) Paragraphs (2) through (4) shall not apply to any class action in which--
(A) the primary defendants are States, State officials, or other governmental entities against
whom the district court may be foreclosed from ordering relief; or
(B) the number of members of all proposed plaintiff classes in the aggregate is less than 100.
(6) In any class action, the claims of the individual class members shall be aggregated to
determine whether the matter in controversy exceeds the sum or value of
$5,000,000, exclusive of interest and costs.
(7) Citizenship of the members of the proposed plaintiff classes shall be determined for
purposes of paragraphs (2) through (6) as of the date of filing of the complaint
or amended complaint, or, if the case stated by the initial pleading is not subject to Federal
jurisdiction, as of the date of service by plaintiffs of an amended pleading,
motion, or other paper, indicating the existence of Federal jurisdiction.
(8) This subsection shall apply to any class action before or after the entry of a class certification
order by the court with respect to that action.
(9) Paragraph (2) shall not apply to any class action that solely involves a claim--
(A) concerning a covered security as defined under 16(f)(3)1 of the Securities Act of 1933 (15
U.S.C. 78p(f)(3)2) and section 28(f)(5)(E) of the Securities Exchange
Act of 1934 (15 U.S.C. 78bb(f)(5)(E));
(B) that relates to the internal affairs or governance of a corporation or other form of business
enterprise and that arises under or by virtue of the laws of the State in
which such corporation or business enterprise is incorporated or organized; or
(C) that relates to the rights, duties (including fiduciary duties), and obligations relating to or
created by or pursuant to any security (as defined under section 2(a)(1) of
the Securities Act of 1933 (15 U.S.C. 77b(a)(1)) and the regulations issued thereunder).
(10) For purposes of this subsection and section 1453, an unincorporated association shall be
deemed to be a citizen of the State where it has its principal place of
business and the State under whose laws it is organized.
(11)(A) For purposes of this subsection and section 1453, a mass action shall be deemed to be a
class action removable under paragraphs (2) through (10) if it otherwise
meets the provisions of those paragraphs.

Downloaded From OutlineDepot.com

(B)(i) As used in subparagraph (A), the term “mass action” means any civil action (except a civil
action within the scope of section 1711(2)) in which monetary relief
claims of 100 or more persons are proposed to be tried jointly on the ground that the plaintiffs'
claims involve common questions of law or fact, except that jurisdiction
shall exist only over those plaintiffs whose claims in a mass action satisfy the jurisdictional
amount requirements under subsection (a).
(ii) As used in subparagraph (A), the term “mass action” shall not include any civil action in
which--
(I) all of the claims in the action arise from an event or occurrence in the State in which the
action was filed, and that allegedly resulted in injuries in that State or in
States contiguous to that State;
(II) the claims are joined upon motion of a defendant;
(III) all of the claims in the action are asserted on behalf of the general public (and not on behalf
of individual claimants or members of a purported class) pursuant to a
State statute specifically authorizing such action; or
(IV) the claims have been consolidated or coordinated solely for pretrial proceedings.
(C)(i) Any action(s) removed to Federal court pursuant to this subsection shall not thereafter be
transferred to any other court pursuant to section 1407, or the rules
promulgated thereunder, unless a majority of the plaintiffs in the action request transfer
pursuant to section 1407.
(ii) This subparagraph will not apply--
(I) to cases certified pursuant to rule 23 of the Federal Rules of Civil Procedure; or
(II) if plaintiffs propose that the action proceed as a class action pursuant to rule 23 of the
Federal Rules of Civil Procedure.
(D) The limitations periods on any claims asserted in a mass action that is removed to Federal
court pursuant to this subsection shall be deemed tolled during the period
that the action is pending in Federal court.
(e) The word “States”, as used in this section, includes the Territories, the District of Columbia,
and the Commonwealth of Puerto Rico.

iii. 28 U.S.C. §1359

a) Parties collusively or improperly joined in order to invoke DJ

§1359. Parties collusively joined or made.


A district court shall not have jurisdiction of a civil action in which any party, by assignment or
otherwise, has been improperly or collusively made or joined to invoke
the jurisdiction of such court.

iv. DIVERSITY REQUIREMENTS MUST BE MET AT TIME SUIT IS FILED!!!


v. Complete Diversity » look to citizenship of parties!

a) Is citizenship of parties completely diverse as required?

vi. How do you determine a party's citizenship?

a) Individual
1. Domicile
(a) Physical Presence
(b) Intent to Remain Indefinitely

Redner v. Sanders [RULE #1: Diversity jurisdiction requires complete diversity between
parties]
Cites Strawbridge v. Curtis » established "complete diversity" requirement
Constitution [Art. III, §2] requires only minimal amount of diversity
Statute [§1332] is interpreted by courts to require "complete diversity"
Congress makes statutes to narrow authority conferred in federal courts under the
Constitution
Constitution = ceiling; sets outer parameters and limits of federal court authority
Statute = narrows scope of federal court authority by giving more specific rules and
regulations on
how to do things
TAKE-AWAY: Congress has the power to tell us what to do and how to do things in a way that
comports with the broader parameter set by the Constitution
RULE #2: Residence ≠ Citizenship » Domicile = Citizenship
Being a resident in another country or state does not equate to domicile » only ONE domicile!
You can have many residences but ONLY ONE DOMICILE at a time

b) Corporation
1. State of Incorporation [Inc.]
2. Principal Place of Business [PPB]
(a) Old Rule: Muscle Test » New Rule: Nerve Center Test
Muscle test: Where the corporation does most of what it does.
Eliminated by S.C. in Hertz v. Friend (2010) »
Nerve center test: Where are the corporate headquarters.

Downloaded From OutlineDepot.com

c) Partnerships and LLCs


1. LLCs are treated like Partnerships » citizenship based on collection of
individuals
(a) Partnership = acquire citizenship of each partner
(b) LLCs = acquire citizenship of each member
2. Belleville Catering v. Champaign [lawyer mistakenly treated Champaign,
LLC like a corporation rather than a partnership]

II. PERSONAL JURISDICTION


A. Basis
i. Long-Arm Statutes » Is there an applicable long-arm statute? Is it constitutional?

a) Step 1: Is PJ permitted under the applicable long-arm statute?


1. If in Federal court » Federal Long-Arm Statute [Rule 4(k)]
2. If in State court » State Long-Arm Statute [equivalent to Rule 4(k)]
b) Step 2: Is long-arm statute constitutional?
1. If long-arm grants SJ [move on to (ii)] » Due Process analysis
2. If long-arm grants GF [move on to (iii)] » Due Process analysis

Gibbons v. Brown [PJ is not permitted by long-arm statute » do not need to go onto due
process and
minimum contacts analysis since long-arm statute is not satisfied]
Gee v. Reingold [no PJ under state long-arm statute]
Mwani v. Bin Laden [look to long-arm statute » look to 5th Amendment for Due Process
analysis
because we are in federal court » 14th Amendment Due Process is for STATE not federal]

What are Long-Arm statutes? Long-Arm statutes create jurisdiction in states of each court.
Limits are defined
by constitution » Two Questions » (1) Does the statute provide PJ? + (2) Is the statute
constitutional?
Federal “Long-Arm Statute” in Rule 4
Rule 4(k) specifies when a federal court may assert personal jurisdiction over a defendant
served under
Rule 4. In this sense it is like a state long-arm statute by which a state legislature specifies the
circumstances in which personal jurisdiction may be exercised by its court.
How does it work? If a state court in the forum state would have PJ over defendant (via
analysis of
state’s long-arm and due process clause), then a federal court in that state does too. [4(k)(1)(A)]
If not, there are other alternatives for a federal court to exercise personal jurisdiction:
Rule 4(k)(1)(C): when nationwide service is authorized by federal statute
Rule 4(k)(2): Federal Question cases: if no state has jurisdiction, serving summons
establishes PJ as long as constitutional; unclear whether aggregate U.S. contacts enough

Rule 4. Summons.
(k) Territorial Limits of Effective Service.
(1) In General. Serving a summons or filing a waiver of service establishes personal jurisdiction
over a defendant:
(A) who is subject to the jurisdiction of a court of general jurisdiction in the state where the
district court is located;
(B) who is a party joined under Rule 14 or 19 and is served within a judicial district of the United
States and not more than 100 miles from where the summons was
issued; or
(C) when authorized by a federal statute.
(2) Federal Claim Outside State-Court Jurisdiction. For a claim that arises under federal law,
serving a summons or filing a waiver of service establishes personal
jurisdiction over a defendant if:
(A) the defendant is not subject to jurisdiction in any state's courts of general jurisdiction; and
(B) exercising jurisdiction is consistent with the United States Constitution and laws.

ii. Specific Jurisdiction

a) Minimum Contacts Test »


1. Applies to individuals and corporate defendants

Downloaded From OutlineDepot.com


2. If a defendant commits an act outside of her state of residence, she will be
subject to specific personal jurisdiction for the “claims arising under the
action”
International Shoe v. Wagon [RULE: Even if defendant is not in the forum state, if the
defendant has minimum contacts in the forum it will not offend traditions of fair play and
substantial justice]
Court says it is doubly fair to hold defendant responsible in a state when the activities in
the state give rise to the cause of action. Non-present defendants must have minimum
contacts with the forum state in order for due process to be met.
Contrast to Pennoyer. Court asks “is it FAIR?” rather than “is it THERE?”

Shoe Spectrum

Contacts None Casual and


isolated

Single or
occasional

Continuous
and systematic

Substantial and
pervasive

Jurisdictional
consequences

None No general
jurisdiction;
No specific
jurisdiction

Specific
jurisdiction

Specific
jurisdiction

General
jurisdiction

NOTE: The closer the ties to the claim, the fewer the contacts needed.
[Example: if you get into a car accident in another state, the claim arises directly out of that
contact, so the
instance is enough to qualify for personal jurisdiction]
McGee v. International Life Insurance
RULE: Contacts are minimal but claim is directly related to cause of action » PJ]
Hanson v. Denckla
RULE: The claim does not arise out of actions consummated in forum state and
defendant corporation did not purposely avail themselves to business in Florida,
obtaining benefits and protection of Florida law. Rather, defendant engaged in business in
PA and then customer [plaintiff] moved to Florida. It was a unilateral move by the
customer to go to Florida » UNILATERAL ACTIVITY = NO PJ!
Court is looking at concept of FAIRNESS.
Contacts may be minimal, but defendant didn’t purposely avail himself of forum
state [like McGee where company sought out business in forum]
Focus shifts to purposeful availment.

Shaffer v. Heitner [TAKE-AWAYS: (1) minimum contacts apply to corporations and


individuals; (2) property is a contact but the claim must arise out of the contact; (3) quasi in
rem doesn’t work anymore]
b) Purposeful Availment [introduced in Hanson]
Defendant must have purposely availed itself of the privilege of conducting activities
within the forum State, thus invoking the benefits and protections of its laws
[Hanson]
Stream of Commerce
“Mere awareness” that stream may sweep goods into forum state is not
enough
O’Connor test: did the defendant design the product specifically for the forum
state, advertise specifically in the forum state, follow restrictions specific to
the forum state? [Asahi]

Downloaded From OutlineDepot.com

World-Wide Volkswagon v. Woodson [no purposeful availment » unilateral]


Asahi v. California Superior Court
Did Defendant Target the Forum state?
Buckeye v. Superior Court [RULE: Manufacturers will be subject to personal
jurisdiction wherever its products are sold and the product is directly related to the
cause of action unless the defendant can prove that the product involved with the
case was not brought to the forum through regular business channels, and there is
some burden to defendant to defend itself in that state]
McIntyre v. Nicastro added new criteria for specific PJ stream-of-commerce cases
RULE: Unless business specifically targeted the forum state, it won’t be
enough for stream of commerce theory of specific jurisdiction.
c) Asahi Fairness Factors (5): » [“traditional notions of fair play and substantial
justice” » court looks to the fairness factors AFTER looking at minimum contacts and
only if the minimum contacts are uncertain]
1. Burden on defendant
2. Interests of the forum state
3. Plaintiff’s interest in obtaining convenient and effective relief
4. Interstate’s interest in obtaining the most efficient resolution of controversies
5. Shared interest of states in furthering fundamental substantive social policies
Example: Burger King [breach of contract case applying fairness factors]
Contract probably not enough for minimum contacts, but it was fair to bring him
to court there since Miami office handled all financial and business affairs with
Michigan franchise.
d) Zippo Test [Website/Internet/E-Commerce SJ]
Scale on Level of Activity [Active --- Interactive ---- Passive]
Active sites: does business and enters contracts on a repeated basis » PJ!
Interactive: exchange of information
Passive: posting information
Example: Abdouch v. Lopez
e) Calder Effects Test » only for intentional tort cases! [Calder v. Jones]
Required Elements for Applying the Calder Effects Test:
1. Defendant committed an intentional tort
2. Defendant injured person in another state
3. Defendant had knowledge injury would occur in another state
TAKE-AWAY: PJ is obtained where the effects of the conduct are felt if the
defendant intended harm there. This is limited to intentional torts.

iii. General Jurisdiction » COA does not need to be related to activities in the state [as in SJ]
Continuous operations within a state are so substantial as to justify suit against it on
causes of action arising from dealings entirely different from these activities.
a) Individual Defendants » Domicile; State of residence
Domicile constitutes general jurisdiction over an individual.
b) Corporation/Business Defendants » PPB or State of Incorporation
Continuous and systematic activity
Corps can be subject to GJ in more than one state [i.e. McDonald’s]

Downloaded From OutlineDepot.com


An equivalent place (equal to an individual’s domicile), aka a place in which
the corporation can be fairly regarded as being “at home”
Goodyear v. Brown [RULE: Minimal sales are insufficient for GJ. A
court may assert GJ over foreign corporations to hear any and all claims
against them when their affiliations with the state are so continuous and
systematic as to render them essentially at home in the forum state.
c) Alternative » systematic and continuous contacts with forum state
Applicable for individual or corporate defendants
Perkins v. Benguet [RULE: GJ if corp has systematic and continuous
presence in the forum state]
Helicopteros [RULE: Mere purchase in a forum state, even if occurring at
regular intervals, are not enough to warrant state’s assertion of personal
jurisdiction over nonresident corporation in cause of action not related to
the purchase transaction. GJ requires SYSTEMATIC and CONTINUOUS
contacts with the forum state]

What about in-person "tag" PJ? » Burnham v. Superior Court [physical presence in state
established PJ]
RULE: States have exclusive jurisdiction over non-residents who are physically within the
state.
REASONING: One who visits another state knowingly assumes risk that the State will exercise
power
over him/her. Although contact is minimal, it gives rise to predictable risks.
Minimum contacts [SJ] necessary to give rise to cause of action relates only to absent
defendants.
TAKE-AWAY: Personal service in a state is still a valid means of obtaining jurisdiction » “Tag
Jurisdiction” [only applicable to individual not corporate defendants]
Tag jurisdiction only applies to individuals » presence in the state is enough to assert
jurisdiction.
However, individuals cannot be tricked into going into the state, presence must be voluntary
and
knowingly (no jurisdiction over witnesses travelling to a state for a trial).
B. Consent
i. Implicit consent: [being within the borders of a state [Burnham]
ii. Express consent:
a. Appointing an agent of service: you thereby consent to service of process in that state and
thus
consent to personal jurisdiction, same with registering to do business if the statute says you
then
consent to service of process.
b. Forum selection clause: You agree beforehand where the lawsuit will happen, constitutes
express consent to personal jurisdiction.
Test for enforceability:
Was the clause negotiable? (if yes, enforceable) If not – consider other factors:
Benefit to the other party
Fair notice
Motive (must not be to discourage lawsuits)

c. Cognovits clause: Shifts the burden. Whoever brings the lawsuit automatically wins.
Defendant
can set aside judgment but their job to prove they did not breach the contract. Only
enforceable
when both parties get a clear benefit out of the clause.
d. Consent can come in the form of pre-litigation agreements [Carnival] or by waiver [when
defendant appears but fails to challenge jurisdiction]
C. Notice
i. Personal Jurisdiction requires BASIS and NOTICE

a) Wuctcher v. Pizzuti [Supreme Court struck down state statute that said court had
jurisdiction over defendants who use their road and service of process just had to be

Downloaded From OutlineDepot.com

served on secretary of state » did not have to notify the defendant thus, the court
overruled the statute on the grounds that is was unconstitutional because it did not
require notice or specify notice to be given to the defendant]

ii. Two-Prong Analysis [Mullane]


Step (1): Does method of notice satisfy applicable law/rule [state or federal]?
State or Federal Rule [Rule 4] must be satisfied » method is important.
Look to state statutes if in state court
Look to Federal Rule [4(e)] if in federal court.
Step (2): Does the notice comply with the Constitution [Due Process]?
Constitutional requirement for notice:
(a) Reasonably calculated and certain to inform; or
(b) If notice is not certain, then choose a method of service that is not substantially
less likely to inform than any other method [choose the best option; take practical
steps towards giving notice to the party]
Sets the minimum amount of notice that a party must get from other party
States cannot fall beneath this standard

iii. Why is Notice and Service a Two-prong Inquiry? Because it must meet BOTH:
State/federal statute or rule requirements and (2) constitutional requirements
Notice needs to:
(1) Reasonably convey what you are notifying them about
(2) Tell them what is coming
(3) Opportunity to present objections (time to be heard/prepare)
Does constitution require defendant get notice?
A mere gesture is not appropriate for due process
Method used needs to be truly desirous of actually informing the defendant
the information

iv. Mullane v. Central Hanover » Notice serves function of creating opportunity to be heard
and must be given in a reasonable period of time. Due Process Clause requires
deprivation of life, liberty, or property by adjudication be preceded by notice. States must
balance burden of serving process with due process fundamental right to be heard.
Notice is NOT actual notice » notice just has to be reasonably calculated to apprise
interested parties of the pendency of the action and afford them an opportunity to
present the objections
Need to give notice a reasonable amount of time before the hearing to give the person
the opportunity to be heard

v. Reasonableness in Due Process [Constitutional] Analysis of Notice:


a) To comply with due process, you must employ method you would use if you really
wanted to inform defendant [thus, this chosen method will be defendable on the
ground that it is in itself reasonably certain to inform the other party]; OR
b) If method is not reasonably certain to inform other party, then you must choose a
method that is NOT substantially less likely to notify other party compared to other
feasible and customary substitutes (any alternative)

Milliken v. Meyer [RULE: adequacy of notice depends upon whether or not the substituted
service of process giving notice was reasonably calculated to give defendant actual notice of
the proceedings and an opportunity to be heard]

Downloaded From OutlineDepot.com

Jones v. Flowers [RULE: although certified mail is usually constitutional, in this case
certified mail was unconstitutional because could've done another alternative method which
was reasonably calculated to give notice]
State had mailed two certified notices to the homeowner at a home that he no long lived
at
Notice was returned back to the state but they did not take any other steps to find him
and state sold home » owner sues to get house back
HOLDING: state should have taken other steps to notify the owner » notice here
violated Due Process because it was not reasonably calculated since there were other
methods that the state could have easily taken to serve notice
TAKE-AWAY: Certified mail returned back to the sender = insufficient notice!

Rule 4. Summons.
(a) Contents; Amendments.
(1) Contents. A summons must:
(A) name the court and the parties;
(B) be directed to the defendant;
(C) state the name and address of the plaintiff's attorney or--if unrepresented--of the plaintiff;
(D) state the time within which the defendant must appear and defend;
(E) notify the defendant that a failure to appear and defend will result in a default judgment
against the defendant for the relief demanded in the complaint;
(F) be signed by the clerk; and
(G) bear the court's seal.
(2) Amendments. The court may permit a summons to be amended.
(b) Issuance. On or after filing the complaint, the plaintiff may present a summons to the clerk
for signature and seal. If the summons is properly completed, the clerk
must sign, seal, and issue it to the plaintiff for service on the defendant. A summons--or a copy
of a summons that is addressed to multiple defendants--must be issued
for each defendant to be served.
(c) Service.
(1) In General. A summons must be served with a copy of the complaint. The plaintiff is
responsible for having the summons and complaint served within the time
allowed by Rule 4(m) and must furnish the necessary copies to the person who makes service.
(2) By Whom. Any person who is at least 18 years old and not a party may serve a summons
and complaint.
(3) By a Marshal or Someone Specially Appointed. At the plaintiff's request, the court may order
that service be made by a United States marshal or deputy marshal or
by a person specially appointed by the court. The court must so order if the plaintiff is
authorized to proceed in forma pauperis under 28 U.S.C. § 1915 or as a seaman
under 28 U.S.C. § 1916.
(d) Waiving Service.
(1) Requesting a Waiver. An individual, corporation, or association that is subject to service
under Rule 4(e), (f), or (h) has a duty to avoid unnecessary expenses of
serving the summons. The plaintiff may notify such a defendant that an action has been
commenced and request that the defendant waive service of a summons. The
notice and request must:
(A) be in writing and be addressed:
(i) to the individual defendant; or
(ii) for a defendant subject to service under Rule 4(h), to an officer, a managing or general
agent, or any other agent authorized by appointment or by law to receive
service of process;
(B) name the court where the complaint was filed;
(C) be accompanied by a copy of the complaint, 2 copies of the waiver form appended to this
Rule 4, and a prepaid means for returning the form;
(D) inform the defendant, using the form appended to this Rule 4, of the consequences of
waiving and not waiving service;
(E) state the date when the request is sent;
(F) give the defendant a reasonable time of at least 30 days after the request was sent--or at
least 60 days if sent to the defendant outside any judicial district of the
United States--to return the waiver; and
(G) be sent by first-class mail or other reliable means.
(2) Failure to Waive. If a defendant located within the United States fails, without good cause,
to sign and return a waiver requested by a plaintiff located within the
United States, the court must impose on the defendant:
(A) the expenses later incurred in making service; and
(B) the reasonable expenses, including attorney's fees, of any motion required to collect those
service expenses.
(3) Time to Answer After a Waiver. A defendant who, before being served with process, timely
returns a waiver need not serve an answer to the complaint until 60 days
after the request was sent--or until 90 days after it was sent to the defendant outside any
judicial district of the United States.
(4) Results of Filing a Waiver. When the plaintiff files a waiver, proof of service is not required
and these rules apply as if a summons and complaint had been served
at the time of filing the waiver.
(5) Jurisdiction and Venue Not Waived. Waiving service of a summons does not waive any
objection to personal jurisdiction or to venue.
(e) Serving an Individual Within a Judicial District of the United States. Unless federal law
provides otherwise, an individual--other than a minor, an incompetent
person, or a person whose waiver has been filed--may be served in a judicial district of the
United States by:
(1) following state law for serving a summons in an action brought in courts of general
jurisdiction in the state where the district court is located or where service is
made; or (2) doing any of the following:
(A) delivering a copy of the summons and of the complaint to the individual personally;
(B) leaving a copy of each at the individual's dwelling or usual place of abode with someone of
suitable age and discretion who resides there; or
(C) delivering a copy of each to an agent authorized by appointment or by law to receive service
of process.
(h) Serving a Corporation, Partnership, or Association. Unless federal law provides otherwise or
the defendant's waiver has been filed, a domestic or foreign
corporation, or a partnership or other unincorporated association that is subject to suit under a
common name, must be served:
(1) in a judicial district of the United States:
(A) in the manner prescribed by Rule 4(e)(1) for serving an individual; or
(B) by delivering a copy of the summons and of the complaint to an officer, a managing or
general agent, or any other agent authorized by appointment or by law to
receive service of process and--if the agent is one authorized by statute and the statute so
requires--by also mailing a copy of each to the defendant; or

Downloaded From OutlineDepot.com

(2) at a place not within any judicial district of the United States, in any manner prescribed by
Rule 4(f) for serving an individual, except personal delivery under
(f)(2)(C)(i).
(k) Territorial Limits of Effective Service.
(1) In General. Serving a summons or filing a waiver of service establishes personal jurisdiction
over a defendant:
(A) who is subject to the jurisdiction of a court of general jurisdiction in the state where the
district court is located;
(B) who is a party joined under Rule 14 or 19 and is served within a judicial district of the United
States and not more than 100 miles from where the summons was
issued; or
(C) when authorized by a federal statute.
(2) Federal Claim Outside State-Court Jurisdiction. For a claim that arises under federal law,
serving a summons or filing a waiver of service establishes personal
jurisdiction over a defendant if:
(A) the defendant is not subject to jurisdiction in any state's courts of general jurisdiction; and
(B) exercising jurisdiction is consistent with the United States Constitution and laws.
III. VENUE
A. Venue restricts places where a suit can be brought even if jurisdiction is proper
Statutory issue: constitution does not mention venue.
Venue can be waived by defendant.
Even if waived, court may not to hear a case if venue is improper for judicial economic
reasons.
There might be a more appropriate venue.

B. 28 U.S.C. §1391
A. (b) Venue in General.
(b)(1) » If all defendants reside in same state, venue is proper in district where any defendant
resides [resides = domicile]
(b)(2) » Venue based on events or omissions giving rise to the claim
(b)(3) » Catchall provision
Venue is authorized in a judicial district where any defendant is subject to personal
jurisdiction (a)(3), or where defendant may be found (b)(3), at the time the action is
commenced only if there is no district where the action can otherwise be brought.
Only applies if there is no district, ANYWHERE in the US where the case can be
brought under sections 1 and 2

B. (c)-(d) Residency » for corporations, aliens, individuals.


§1391. Venue generally
(b) Venue in general.--A civil action may be brought in--
(1) a judicial district in which any defendant resides, if all defendants are residents of the State
in which the district is located;
(2) a judicial district in which a substantial part of the events or omissions giving rise to the
claim occurred, or a substantial part of property that is the subject of the
action is situated; or
(3) if there is no district in which an action may otherwise be brought as provided in this
section, any judicial district in which any defendant is subject to the court's
personal jurisdiction with respect to such action.
(c) Residency.--For all venue purposes--
(1) a natural person, including an alien lawfully admitted for permanent residence in the United
States, shall be deemed to reside in the judicial district in which that
person is domiciled;
(2) an entity with the capacity to sue and be sued in its common name under applicable law,
whether or not incorporated, shall be deemed to reside, if a defendant, in
any judicial district in which such defendant is subject to the court's personal jurisdiction with
respect to the civil action in question and, if a plaintiff, only in the
judicial district in which it maintains its principal place of business; and
(3) a defendant not resident in the United States may be sued in any judicial district, and the
joinder of such a defendant shall be disregarded in determining where the
action may be brought with respect to other defendants.
(d) Residency of corporations in States with multiple districts.--For purposes of venue under this
chapter, in a State which has more than one judicial district and in
which a defendant that is a corporation is subject to personal jurisdiction at the time an action
is commenced, such corporation shall be deemed to reside in any district
in that State within which its contacts would be sufficient to subject it to personal jurisdiction if
that district were a separate State, and, if there is no such district, the
corporation shall be deemed to reside in the district within which it has the most significant
contacts.
IV. DECLINING JURISDICTION: TRANSFER AND FORUM-NON
A. Statutory Rule » applies only to the federal courts [allows transfer among federal
districts]
Transfer = applies to federal courts only » allow cases to be moved around the
country for the convenience of parties and the witnesses in the interest of justice.
28 U.S.C. §1404: Change of Venue [Transfer]

Downloaded From OutlineDepot.com

Venue is INCONVENIENT » use law of state where case is initially brought

§1404. Change of venue


(a) For the convenience of parties and witnesses, in the interest of justice, a district court may
transfer any civil action to any other district or division where
it might have been brought.
(b) Upon motion, consent or stipulation of all parties, any action, suit or proceeding of a civil
nature or any motion or hearing thereof, may be transferred, in
the discretion of the court, from the division in which pending to any other division in the same
district. Transfer of proceedings in rem brought by or on
behalf of the United States may be transferred under this section without the consent of the
United States where all other parties request transfer.
(c) A district court may order any civil action to be tried at any place within the division in which
it is pending.
(d) As used in this section, the term “district court” includes the District Court of Guam, the
District Court for the Northern Mariana Islands, and the District
Court of the Virgin Islands, and the term “district” includes the territorial jurisdiction of each
such court
28 U.S.C. §1406(a): Cure or Waiver of Defects
Move to dismiss under 1406(a) only if venue is "wrong" or "improper"
Venue is IMPROPER » use law of state where case is transferred to

§1406. Cure or waiver of defects


(a) The district court of a district in which is filed a case laying venue in the wrong division or
district shall dismiss, or if it be in the interest of justice, transfer such
case to any district or division in which it could have been brought.
(b) Nothing in this chapter shall impair the jurisdiction of a district court of any matter involving
a party who does not interpose timely and sufficient objection to the
venue.
(c) As used in this section, the term “district court” includes the District Court of Guam, the
District Court for the Northern Mariana Islands, and the District Court of
the Virgin Islands, and the term “district” includes the territorial jurisdiction of each such court.
B. Common Law Rule » applies to state and federal courts [state statute = §1404(a)]
Forum Non Conveniens = a common law doctrine. Used in federal and state courts to send
case to a foreign country or another state.
Gulf Oil Analysis
Private Interest Factors:
Relative ease of access to sources of proof
Availability of compulsory process for witnesses who are not willing to come forward
Cost for willing witnesses
Ability to view premises/scene of the incident
Practical costs and ease of trial
Public Interest Factors:
Court congestion
Local issues should be settled at home
Court’s familiarity with applicable law
Avoiding conflicts of law/application of foreign law
Burdening citizens in an unrelated forum with jury duty
C. Forum-non v. Transfer
Forum-non is used in federal court when you want to transfer to a foreign jurisdiction. Also
used in state court when you want to transfer to another state’s courts. 1404 and 1406 are
used to transfer from one district to another in federal court (or to a different division
within a district).
Forum-non is considered a dismissal, SOL will continue to run but parties will often stipulate
to that it will not be an issue.
Transfer is not a dismissal, no SOL issue, no need to re-file.

D. TAKE-AWAY: If court does not have SMJ or PJ, it MUST dismiss » If PJ and venue are
proper, the court MAY dismiss thus, forum-non is discretionary and up to the court.

V. JOINDER OF CLAIMS
A. Rule 18: Joinder of Claims
A single party can join any and all claims he has against opposing party
Proper Joinder ≠ Proper Jurisdiction » still need proper SMJ and PJ

Downloaded From OutlineDepot.com


Rule 18(a) » A party seeking relief from an opposing party MAY join with his
original claim ANY additional claim he has against that opposing party
Additional claims that arise out of the same transaction or occurrence that are
NOT asserted may later be barred by claim preclusion.

Rule 18. Joinder of Claims


(a) In General. A party asserting a claim, counterclaim, crossclaim, or third-party claim may join,
as independent or alternative claims, as many claims as it
has against an opposing party.
(b) Joinder of Contingent Claims. A party may join two claims even though one of them is
contingent on the disposition of the other; but the court may
grant relief only in accordance with the parties' relative substantive rights. In particular, a
plaintiff may state a claim for money and a claim to set aside a
conveyance that is fraudulent as to that plaintiff, without first obtaining a judgment for the
money.
B. Rule 13: Claims by Defendants [Counterclaims and Crossclaims]
i. 13(b) » Permissive Counterclaims

a) Defendant can bring as a permissive counterclaim, any claim


(related or unrelated) against a plaintiff.

ii. 13(a) » Compulsory Counterclaims

a) Defendant can bring as a compulsory counterclaim, any claim that arises


out of the transaction or occurrence that is the subject matter of the plaintiff's
claim.
Rule 13. Counterclaim and Crossclaim
(a) Compulsory Counterclaim.
(1) In General. A pleading must state as a counterclaim any claim that--at the time of its service-
-the pleader has against an opposing party if the claim:
(A) arises out of the transaction or occurrence that is the subject matter of the opposing party's
claim; and
(B) does not require adding another party over whom the court cannot acquire jurisdiction.
(2) Exceptions. The pleader need not state the claim if:
(A) when the action was commenced, the claim was the subject of another pending action; or
(B) the opposing party sued on its claim by attachment or other process that did not establish
personal jurisdiction over the pleader on that claim, and the pleader does
not assert any counterclaim under this rule.
(b) Permissive Counterclaim. A pleading may state as a counterclaim against an opposing party
any claim that is not compulsory.
(g) Crossclaim Against a Coparty. A pleading may state as a crossclaim any claim by one party
against a coparty if the claim arises out of the transaction or
occurrence that is the subject matter of the original action or of a counterclaim, or if the claim
relates to any property that is the subject matter of the original action. The
crossclaim may include a claim that the coparty is or may be liable to the cross-claimant for all
or part of a claim asserted in the action against the cross-claimant.
C. Implications of Compulsory Counterclaims [under 13(b)]
i. Compulsory counterclaims must be brought or you risk losing it » argument is
that
defendants who fail to bring compulsory counterclaims waive it
2. If Compulsory counterclaim is brought, it is covered under supplemental
jurisdiction [§1367]
a) Penalty for omitting a counterclaim that is later held to be compulsory
b) No penalty for including a counterclaim that is held not to be compulsory
[that is why defense counsel will often bring all possible counterclaims]

ARISING OUT OF THE SAME TRANSACTION OR OCCURRENCE


Four Tests to determine if claim is compulsory [arises out of the same transaction or
occurrence]:
Are the issues of fact or law raised by the claim and counterclaims largely the same?
Would res judicata bar a subsequent suit on defendant’s claim absent the compulsory
counter
claim? (So much already litigated in original action that it can’t be brought in a separate suit)
Will substantially the same evidence support or refute plaintiff’s claims as well as
defendant’s counterclaims?
Logical Relationship Test – Is there any logical relation between the claim and the
counterclaim? (Most commonly used)
An affirmative answer to any of these tests indicates that the claim is compulsory
Linking Rule 13(a)(g) with Rule 18(a)
Step 1) Claim must arise out of the transaction or occurrence [Rule 13]

Downloaded From OutlineDepot.com

Step 2) Once defendant joins claim arising out of same T/O, then he can add on more claims
that
are not related [Rule 18]
A defendant can only join a claim against another defendant [cross claim] if it arises out of
the same t/o. After there is one that meets the t/o test, he can assert more. D CANNOT assert
an unrelated claim against another D without a cross claim first. By definition, cross-claims
always must arise out of the same t/o.
VI. JOINDER OF PARTIES
A. Rule 20. Permissive Joinder of Parties.
i. 20(a)(1) » Joinder of Parties by Plaintiffs.
ii. 20(a)(2) » Joinder of Parties by Defendants.

Rule 20. Permissive Joinder of Parties


(a) Persons Who May Join or Be Joined.
(1) Plaintiffs. Persons may join in one action as plaintiffs if:
(A) they assert any right to relief jointly, severally, or in the alternative with respect to or arising
out of the same transaction, occurrence, or series of transactions or
occurrences; and
(B) any question of law or fact common to all plaintiffs will arise in the action.
(2) Defendants. Persons--as well as a vessel, cargo, or other property subject to admiralty
process in rem--may be joined in one action as defendants if:
(A) any right to relief is asserted against them jointly, severally, or in the alternative with
respect to or arising out of the same transaction, occurrence, or series of
transactions or occurrences; and
(B) any question of law or fact common to all defendants will arise in the action.
B. Rule 21. Misjoinder and Nonjoinder of Parties.
i. “Misjoinder of parties is not a ground for dismissing an action. On motion or on
its own, the court may at any time, on just terms, add or drop a party. The court
may also sever any claim against a party.” » If parties are improperly joined, cases
can be severed
Rule 21. Misjoinder and Nonjoinder of Parties
Misjoinder of parties is not a ground for dismissing an action. On motion or on its own, the
court may at any time, on just terms, add or drop a party. The court may also
sever any claim against a party.
C. Rule 14. Third-Party Practice.
i. 14(a) » a defendant may assert a claim against anyone not a party to the original
action if that third party's liability is in some way dependent upon the outcome of
the original action
ii. This rule exemplifies this idea of "impleader"
a. Impleader = a procedure by which a defendant in a civil action may assert a
claim against a third-party for that party’s liability to the defendant, such as a
claim by a defendant for contribution or indemnification.
b. 14 does not create derivative liability, but instead, offers a procedural channel
through which that liability can be asserted in the main suit rather than in a
separate one

Rule 14. Third-Party Practice


(a) When a Defending Party May Bring in a Third Party.
(1) Timing of the Summons and Complaint. A defending party may, as third-party plaintiff, serve
a summons and complaint on a nonparty who is or may be liable to it
for all or part of the claim against it. But the third-party plaintiff must, by motion, obtain the
court's leave if it files the third-party complaint more than 14 days after
serving its original answer.
(2) Third-Party Defendant's Claims and Defenses. The person served with the summons and
third-party complaint--the “third-party defendant”:
(A) must assert any defense against the third-party plaintiff's claim under Rule 12;
(B) must assert any counterclaim against the third-party plaintiff under Rule 13(a), and may
assert any counterclaim against the third-party plaintiff under Rule 13(b) or
any crossclaim against another third-party defendant under Rule 13(g);
(C) may assert against the plaintiff any defense that the third-party plaintiff has to the plaintiff's
claim; and
(D) may also assert against the plaintiff any claim arising out of the transaction or occurrence
that is the subject matter of the plaintiff's claim against the third-party
plaintiff.
(3) Plaintiff's Claims Against a Third-Party Defendant. The plaintiff may assert against the third-
party defendant any claim arising out of the transaction or occurrence
that is the subject matter of the plaintiff's claim against the third-party plaintiff. The third-party
defendant must then assert any defense under Rule 12 and any
counterclaim under Rule 13(a), and may assert any counterclaim under Rule 13(b) or any
crossclaim under Rule 13(g).
(4) Motion to Strike, Sever, or Try Separately. Any party may move to strike the third-party
claim, to sever it, or to try it separately.

Downloaded From OutlineDepot.com

(5) Third-Party Defendant's Claim Against a Nonparty. A third-party defendant may proceed
under this rule against a nonparty who is or may be liable to the third-
party defendant for all or part of any claim against it.

(6) Third-Party Complaint In Rem. If it is within the admiralty or maritime jurisdiction, a third-
party complaint may be in rem. In that event, a reference in this rule to
the “summons” includes the warrant of arrest, and a reference to the defendant or third-party
plaintiff includes, when appropriate, a person who asserts a right under
Supplemental Rule C(6)(a)(i) in the property arrested.
(b) When a Plaintiff May Bring in a Third Party. When a claim is asserted against a plaintiff, the
plaintiff may bring in a third party if this rule would allow a
defendant to do so.
VII. SUPPLEMENTAL JURISDICTION
A. Constitutional Component » maximum reach of federal jurisdiction [United Mine v.
Gibbs]
B. Statutory Component » 28 U.S.C. §1367
i. Narrower scope than the Constitutional aspect; broadens federal jurisdiction so
federal courts can hear and exercise jurisdiction over (cross)claims that are so related to
an initial claim but would not ordinarily fall within the court's subject matter jurisdiction
C. 1367(a) » Must be satisfied in both Diversity and FQ cases
i. Supplemental jurisdiction extends to cover claims [in which federal courts do not
have original SMJ over] if the claims arises out of the "same case or controversy" »
common nucleus of operative fact
ii. Does second claim w/ no independent basis for SMJ arise out of the same "case or
controversy" out of an initial claim w/ independent basis for SMJ?
a. YES = Supplemental Jurisdiction!
b. NO = No Supplemental Jurisdiction!
D. 1367(b) » Must be satisfied ONLY in Diversity cases [(b) does not apply to FQ]
i. If 1367(a) is satisfied, cases ONLY in federal court for diversity purposes will also
need to satisfy 1367(b) in order to establish supplemental jurisdiction!
ii. If second claim [that is proper under joinder of claims] destroys diversity, then (b)
kicks in and supplemental jurisdiction will not extend to cover the second claim
iii. (b) limits court's ability to exercise supplemental jurisdiction over claims in cases
where sole basis for SMJ is diversity
iv. Main Purpose of 1367(b):
a. Stop "gamesmanship" by parties » prevents original plaintiff from
commencing a lawsuit that initially meets requirement of complete diversity
but then later on tries to bring in a nondiverse party by bringing a
supplemental jurisdiction claim against that nondiverse party;
b. Prevent plaintiffs who are basically using supplemental jurisdiction to get
around the complete diversity requirement by bringing a non-diverse
defendant under joinder of parties later on in the lawsuit

E. 1367(c) » Courts may Decline to Exercise Supplemental Jurisdiction on Four (4) Grounds:
i. Claim raises novel/complex state law issue
ii. Claim substantially predominates over claim of original, independent SMJ
iii. Federal court dismissed claims to which it had independent SMJ over
iv. Other compelling reasons that arise under exceptional circumstances

§1367. Supplemental Jurisdiction.


(a) Except as provided in subsections (b) and (c) or as expressly provided otherwise by Federal
statute, in any civil action of which the district courts have original
jurisdiction, the district courts shall have supplemental jurisdiction over all other claims that are
so related to claims in the action within such original jurisdiction that
they form part of the same case or controversy under Article III of the United States
Constitution. Such supplemental jurisdiction shall include claims that involve the
joinder or intervention of additional parties.
(b) In any civil action of which the district courts have original jurisdiction founded solely on
section 1332 of this title, the district courts shall not have supplemental
jurisdiction under subsection (a) over claims by plaintiffs against persons made parties under
Rule 14, 19, 20, or 24 of the Federal Rules of Civil Procedure, or over
claims by persons proposed to be joined as plaintiffs under Rule 19 of such rules, or seeking to
intervene as plaintiffs under Rule 24 of such rules, when exercising
supplemental jurisdiction over such claims would be inconsistent with the jurisdictional
requirements of section 1332.
(c) The district courts may decline to exercise supplemental jurisdiction over a claim under
subsection (a) if--
(1) the claim raises a novel or complex issue of State law,

Downloaded From OutlineDepot.com


(2) the claim substantially predominates over the claim or claims over which the district court
has original jurisdiction,
(3) the district court has dismissed all claims over which it has original jurisdiction, or
(4) in exceptional circumstances, there are other compelling reasons for declining jurisdiction.
(d) The period of limitations for any claim asserted under subsection (a), and for any other
claim in the same action that is voluntarily dismissed at the same time as or
after the dismissal of the claim under subsection (a), shall be tolled while the claim is pending
and for a period of 30 days after it is dismissed unless State law provides
for a longer tolling period.
(e) As used in this section, the term “State” includes the District of Columbia, the
Commonwealth of Puerto Rico, and any territory or possession of the United States.
VIII.REMOVAL JURISDICTION
A. 28 U.S.C. §1441. Removal of Civil Actions.
If plaintiff files suit in state court but could have filed in federal court, defendant can remove
1441(a) » unless federal law says otherwise, any civil action initially brought in state court
may be
removed to federal court if federal court has “original jurisdiction” (SMJ) over lawsuit.
Generally, a defendant can remove to federal court any suit which the plaintiff could have
commenced in federal court (because federal court would have had SMJ over it)
1441(a) provides that a case may be removed “by the defendant or the defendants”; the
Supreme Court has interpreted this to mean original defendants, so that the original plaintiff
(now turned counterclaim defendant) cannot have the case removed.
1441(b)(2) » limitation on removal for diversity cases » citizenship limitation
If removal to federal court is solely based on SMJ for diversity purposes, removal will only be
allowed if none of the defendants are citizens of the state in which the plaintiff brought the suit
If any defendant is a citizen of that state where the plaintiff filed the suit, then removal to
federal court will not be permitted

1441(c)(2) » All defendants in the case must join in, or consent to, the removal.
§1441. Removal of civil actions
(a) Generally.--Except as otherwise expressly provided by Act of Congress, any civil action
brought in a State court of which the district courts of the United States
have original jurisdiction, may be removed by the defendant or the defendants, to the district
court of the United States for the district and division embracing the place
where such action is pending.
(b) Removal based on diversity of citizenship.--(1) In determining whether a civil action is
removable on the basis of the jurisdiction under section 1332(a) of this
title, the citizenship of defendants sued under fictitious names shall be disregarded.
(2) A civil action otherwise removable solely on the basis of the jurisdiction under section
1332(a) of this title may not be removed if any of the parties in interest
properly joined and served as defendants is a citizen of the State in which such action is
brought.
(c) Joinder of Federal law claims and State law claims.--(1) If a civil action includes--
(A) a claim arising under the Constitution, laws, or treaties of the United States (within the
meaning of section 1331 of this title), and
(B) a claim not within the original or supplemental jurisdiction of the district court or a claim
that has been made nonremovable by statute, the entire action may be
removed if the action would be removable without the inclusion of the claim described in
subparagraph (B).
(2) Upon removal of an action described in paragraph (1), the district court shall sever from the
action all claims described in paragraph (1)(B) and shall remand the
severed claims to the State court from which the action was removed. Only defendants against
whom a claim described in paragraph (1)(A) has been asserted are
required to join in or consent to the removal under paragraph (1).

B. 28 U.S.C. §1446. Procedure for Removal.


Federal statute that sets forth the procedural requirements for removal of a civil action from
state to
federal court
1446(a) » the party or parties seeking removal must file (in the appropriate U.S. District Court
and division) a document called a “notice of removal,” which must contain “a short and plain
statement of the grounds for removal” plus “a copy of all process pleadings and orders” in the
state court action which were served on the defendant(s)
1446(b) » time frame = defendant has 30 days to file notice of removal after defendant's
receipt
of the initial pleading (summons; complaint)
(b)(3) » If the case is not removable initially, based on the plaintiff’s complaint, but the
plaintiff later amends the complaint or joins additional parties such that the case becomes
removable, the defendant has 30 days after being served with the papers so indicating to file
a notice of removal.

Downloaded From OutlineDepot.com

1446(c) » if removal is based solely on diversity purposes, can remove within the 1 year-
period
after complaint is filed; cannot remove after 1-year period is up
1446(d) » after filing removal, defendants must give notice to all removed “adverse” parties
and
file notice with state court clerk;
“Promptly after” the notice of removal is filed in the appropriate federal district court, the
parties seeking removal must “give written notice thereof to all adverse parties and... file a
copy of the notice” with the clerk of the state court where the lawsuit was commenced.
After filing copy with state court clerk, the State court can no longer proceed unless case
is remanded.
§1446. Procedure for Removal of Civil Actions.
(a) Generally.--A defendant or defendants desiring to remove any civil action from a State court
shall file in the district court of the United States for the district and
division within which such action is pending a notice of removal signed pursuant to Rule 11 of
the Federal Rules of Civil Procedure and containing a short and plain
statement of the grounds for removal, together with a copy of all process, pleadings, and
orders served upon such defendant or defendants in such action.
(b) Requirements; generally.--(1) The notice of removal of a civil action or proceeding shall be
filed within 30 days after the receipt by the defendant, through service
or otherwise, of a copy of the initial pleading setting forth the claim for relief upon which such
action or proceeding is based, or within 30 days after the service of
summons upon the defendant if such initial pleading has then been filed in court and is not
required to be served on the defendant, whichever period is shorter.
(2)(A) When a civil action is removed solely under section 1441(a), all defendants who have
been properly joined and served must join in or consent to the removal of
the action.
(B) Each defendant shall have 30 days after receipt by or service on that defendant of the initial
pleading or summons described in paragraph (1) to file the notice of
removal.
(C) If defendants are served at different times, and a later-served defendant files a notice of
removal, any earlier-served defendant may consent to the removal even
though that earlier-served defendant did not previously initiate or consent to removal.
(3) Except as provided in subsection (c), if the case stated by the initial pleading is not
removable, a notice of removal may be filed within 30 days after receipt by the
defendant, through service or otherwise, of a copy of an amended pleading, motion, order or
other paper from which it may first be ascertained that the case is one
which is or has become removable.
(c) Requirements; removal based on diversity of citizenship.--(1) A case may not be removed
under subsection (b)(3) on the basis of jurisdiction conferred by
section 1332 more than 1 year after commencement of the action, unless the district court
finds that the plaintiff has acted in bad faith in order to prevent a defendant
from removing the action.
(2) If removal of a civil action is sought on the basis of the jurisdiction conferred by section
1332(a), the sum demanded in good faith in the initial pleading shall be
deemed to be the amount in controversy, except that--
(A) the notice of removal may assert the amount in controversy if the initial pleading seeks--
(i) nonmonetary relief; or
(ii) a money judgment, but the State practice either does not permit demand for a specific sum
or permits recovery of damages in excess of the amount demanded; and
(B) removal of the action is proper on the basis of an amount in controversy asserted under
subparagraph (A) if the district court finds, by the preponderance of the
evidence, that the amount in controversy exceeds the amount specified in section 1332(a).
(3)(A) If the case stated by the initial pleading is not removable solely because the amount in
controversy does not exceed the amount specified in section 1332(a),
information relating to the amount in controversy in the record of the State proceeding, or in
responses to discovery, shall be treated as an ‘other paper’ under
subsection (b)(3).
(B) If the notice of removal is filed more than 1 year after commencement of the action and the
district court finds that the plaintiff deliberately failed to disclose the
actual amount in controversy to prevent removal, that finding shall be deemed bad faith under
paragraph (1).
(d) Notice to adverse parties and State court.--Promptly after the filing of such notice of
removal of a civil action the defendant or defendants shall give written
notice thereof to all adverse parties and shall file a copy of the notice with the clerk of such
State court, which shall effect the removal and the State court shall proceed
no further unless and until the case is remanded.
C. 28 U.S.C. §1447. Procedure after Removal.
Rules for challenging removal after case has been removed
Plaintiff's primary recourse is to make a motion before the federal court where the case was
removed
and ask it to remand the case back to the state court
Generally, there are two bases for remanding the case back to the state court:
Lack of federal subject-matter jurisdiction;
Procedural defects, like missing the deadline to file a notice of removal, or where the
“forum-defendant” rule applies.

File "NOTICE" for Removal v. File "MOTION" for Remand (post-removal)


Time requirement for remand is that it must be 30 days after the filing of notice of removal
No time requirement if plaintiff’s reason for remand is lack of subject matter jurisdiction in
the
federal court [because SMJ is not waivable so whenever it appears case is improperly in federal
court, plaintiff can file a motion to remand by reason of absent SMJ]
1447(c) » if the reason is lack of SMJ, then the plaintiff may move to remand the case at any
time
until a final judgment is entered

Downloaded From OutlineDepot.com

If the defect is procedural, then plaintiff has only 30 days after the notice of removal is filed to
move to remand, or he will forever lose the right to contest removal on this basis.
1447(d) » orders of remand are generally not reviewable on appeal. Statute does not say that
orders
refusing remand are not reviewable.
Caterpillar Inc. v. Lewis [court grants review of district court’s order of denial as to the
plaintiff’s objection to the removal and plaintiff’s motion to remand the case to state court]
1447(e) » if after removal the plaintiff seeks to join additional defendants whose joinder
would
destroy subject matter jurisdiction, the court may deny joinder, or permit joinder and remand
the
action to the State court.
Alternative method for plaintiff who wants to get case remanded to state court
If the plaintiff seeks to join additional defendants in such a way as to destroy SMJ [i.e.
adding a nondiverse defendant in a diversity-only case], then the federal court can do one of
two things: (1) refuse to permit the joinder, or (2) remand the action to the state court.

§1447. Procedure after removal generally


(a) In any case removed from a State court, the district court may issue all necessary orders and
process to bring before it all proper parties whether served by process
issued by the State court or otherwise.
(b) It may require the removing party to file with its clerk copies of all records and proceedings
in such State court or may cause the same to be brought before it by writ
of certiorari issued to such State court.
(c) A motion to remand the case on the basis of any defect other than lack of subject matter
jurisdiction must be made within 30 days after the filing of the notice of
removal under section 1446(a). If at any time before final judgment it appears that the district
court lacks subject matter jurisdiction, the case shall be remanded. An
order remanding the case may require payment of just costs and any actual expenses, including
attorney fees, incurred as a result of the removal. A certified copy of the
order of remand shall be mailed by the clerk to the clerk of the State court. The State court may
thereupon proceed with such case.
(d) An order remanding a case to the State court from which it was removed is not reviewable
on appeal or otherwise, except that an order remanding a case to the State
court from which it was removed pursuant to section 1442 or 1443 of this title shall be
reviewable by appeal or otherwise.
(e) If after removal the plaintiff seeks to join additional defendants whose joinder would
destroy subject matter jurisdiction, the court may deny joinder, or permit
joinder and remand the action to the State court.
IX. CLAIM PRECLUSION
A. "Res judicata" = prevents a case from being brought again; doctrine prohibits the same
parties
from re-litigating the same cause of action after there has been a final judgment on the merits.
B. Four (4) Required Elements of Claim Preclusion:
i. Same cause of action/claim
a) Broad Test = Frier v. City of Vandalia, majority opinion [follows broad
Restatement test but says even the narrow test would be met]
1. Restatement (Second) of Judgments §24 » precludes claims that arise
out of all or any part of the transactions or series of connected
transactions
2. Restatement [broad] definition of claims: transaction or series of
connected transactions that arise out of the same claim depends on the
following considerations: (1) whether facts are related in time, space,
origin or motivation; (2) whether facts form a convenient trial unit; and
(3) whether treatment of factual groupings as a unit meet the parties'
expectations or business understanding or usage
b) Narrow Test = Frier v. City of Vandalia, concurring opinion [Illinois law
does not abide by the broad test in the Restatement but rather it applies a
narrower definition where the parties and the cause of action must be
identical; causes of action are identical where the evidence necessary to
sustain a second verdict would sustain the first, or where the causes of action
are based on an identical core of operative facts]

ii. Judgment on the merits


a) 41(b) » Definition of "on the merits" » any dismissal = final judgment "on the
merits" except for dismissals for lack of jurisdiction, improper venue, or
failure to join party under Rule 19

Downloaded From OutlineDepot.com

iii. Final judgment


a) Ison v. Thomas [A plaintiff who has sued for a part of an entire demand is not
permitted to later sue for the remaining portion in another action. A final
judgment on a plaintiff’s claim extinguishes all rights of the plaintiff to
remedies against the defendant with respect to all or any part of the
transaction, or series of connected transactions, out of which the action arose.
Restate. §24]
b) 60(b)(5) » If judgment is vacated on appeal, then you can bring second claim
iv. Same parties
a) Parties must be exactly the same [contrast to issue preclusion]
C. SMJ in relation to state preclusion laws ["court of competent jurisdiction"]
i. Gargallo v. Merrill Lynch et al. » (1) whether the original dismissal was "on the
merits" for purposes and claim preclusion; and (2) how the jurisdictional defect in the
first proceeding ought to impact/affect claim preclusion
a) Determine whether a prior state court judgment has preclusive effect in a
federal court
i. Look to 28 U.S.C. §1738 [Full Faith & Credit Clause] which requires
a federal court to give a state judgment the same preclusive effect such
judgment would have in a state court
b) Determine whether the state court would give preclusive effect to a state court
judgment upon a cause of action over which the state court has no SMJ over
i. What does the state law say?
a. In Ohio, if a final judgment is rendered by a court of that state
that has no SMJ to hear that case since it is based upon a COA
(federal securities laws) that is within the exclusive jurisdiction
of federal courts, then it will have no claim preclusive effect
[following the position of the Restatement (Second) of
Judgments » no SMJ = no claim preclusion]

Rule 41. Dismissal of Actions.


(b) Involuntary Dismissal; Effect.
If the plaintiff fails to prosecute or to comply with these rules or a court order, a defendant may
move to dismiss the action or any claim against it. Unless the
dismissal order states otherwise, a dismissal under this subdivision (b) and any dismissal not
under this rule--except one for lack of jurisdiction, improper
venue, or failure to join a party under Rule 19--operates as an adjudication on the merits.
Rule 60(b) Grounds for Relief from a Final Judgment, Order, or Proceeding.
On motion and just terms, the court may relieve a party or its legal representative from a final
judgment, order, or proceeding for the following reasons: -- (5)
the judgment has been satisfied, released or discharged; it is based on an earlier judgment that
has been reversed or vacated; or applying it prospectively is no
longer equitable.
28 U.S.C. §1738 [Full Faith & Credit Clause]
requires federal court to give a state court judgement the same preclusive effect such
judgement would have in state court.
X. ISSUE PRECLUSION

A. "Collateral estoppel" = Prevents an issue from being revisited a later case; under this
doctrine,
if an issue is actually litigated, decided, and essential to the decision, the losing party may not
re-litigate such issue in a subsequent proceeding.
B. Only precludes plaintiff from re-litigation issues that were actually litigated and decided in a
prior action. If an issue could have been raised in the first case but was not explicitly raised
and decided, it can be brought in a subsequent action.
C. Required Elements of Issue Preclusion:
i. Same issue of law/fact
ii. Issue is actually litigated and decided [not by default]

Downloaded From OutlineDepot.com

a) Illinois Central Gulf Railroad v. Parks [issue of contributory negligence on the part of
one of the defendants (Jessie) was never actually decided and litigated thus, issue
preclusion did not apply » defendant's contributory negligence was not precluded from
second case because there was not a final judgment or decision on the particular issue
that defendant (Jessie) is now trying to bring up in second case (Jessie's suit against
Railroad; Jessie v. Illinois Central Gulf Railroad]
iii. Final Judgment
iv. Decision was necessary to original judgement
v. Parties can be same or different [but only can bring issue preclusion against the loser!]
a) Parklane Hosiery Co v. Shore » Can only use offensive/defensive issue preclusion
against LOSER of the first case; winner of the first case cannot use

i. Defensive Use of Issue Preclusion = Collateral estoppel is defensive if


a defendant seeks to rely on a prior finding to defeat a plaintiff’s claim
against it in a later case
1. Party cannot invoke issue preclusion against non-party to first
suit [would violate non-party’s due process]
2. Gives plaintiff strong incentive to join all potential defendants in
the first action if possible

ii. Offensive Use of Issue Preclusion = Collateral estoppel is offensive if


a plaintiff seeks to rely on a prior finding to hold the defendant liable in
a later case.
1. Most courts say nonparty can invoke issue preclusion against
party to 1st suit
2. Creates opposite incentive » ‘Wait and see’ [learn from first
case]
a. Hurts judicial economy
b. Unfair to defendant » it gives opportunity to sandbag
defendant
c. Inconsistent judgments
d. Second action may afford new procedural opportunities
XI. THE ERIE DOCTRINE: WHAT LAW APPLIES WHEN YOU ARE IN

FEDERAL COURT FOR DIVERSITY PURPOSES?

A. Swift v. Tyson (1842) » Pre-Erie doctrine


FACTS: Breach of contract claim brought in federal court for diversity
ISSUE: What law do we apply when we are in federal court for diversity purposes but the the
case involves
state claims? » Justice Story says to look at the RDA [tells us what law we should apply here]
28 U.S.C. §1652. Rules of Decision Act.
Take-Away Point » State law applies in federal courts of the US in cases where they apply [i.e.
diversity cases] unless the Constitution, treaties or Acts of Congress otherwise provides
§1652. State laws as rules of decision.
The laws of the several states, except where the Constitution or treaties of the United States or
Acts of Congress otherwise require or
provide, shall be regarded as rules of decision in civil actions in the courts of the United States,
in cases where they apply.
Justice interpreted RDA [codified in statute §1652] to mean we do not need to follow New
York case
law decisions because it is common law not statutory law
He says that the “laws” referred to in the RDA only means statutory law
Since no state statute applicable » do not need to follow state law at all

Downloaded From OutlineDepot.com

Justice Story says in these types of cases of general law where there is no state statute to
apply,
federal courts have the freedom to choose what the best law is and what law should be applied
TAKE-AWAY POINT: Follow state statutory law in diversity cases, but if there is only common
law
then the federal judge can survey all the laws and decide what he thinks is best
If there is a state statute on point, then it must be followed » absent a state statute, federal
judge is
free to look at all laws and decide which is best to apply
Essentially, federal judges can do whatever they want and are free to decide what is best law
to
apply because they were not bound to state [common] law on particular "general" issues
B. Erie v. Tompkins [Constitutional basis for overturning Swift rule]
FACTS: plaintiff walking along railroad in PA; train passes and train door knocks him down;
arm severed.
PROCEDURAL HISTORY: plaintiff sues railroad in federal court [Southern District Ct. of NY] on
diversity grounds after plaintiff's lawyers realizes that the plaintiff has a better chance of
winning in federal
court than state court because federal court won't have to follow the PA state common law
rule on
negligence [defendant only will be found liable to plaintiff trespasser for "wanton" negligence
not
"ordinary" negligence thus, there is a much higher standard in state common law rules which
makes it less
likely that the plaintiff will win » as a result, plaintiff's lawyer sues in federal court hoping they
will apply
the ordinary negligence "general law" rather than the "wanton negligence" rule in PA state
common law so
plaintiff has a better chance of winning]
Judge ignored state law [wanton negligence] » instead, applied "general law" [ordinary
negligence]
Jury returned verdict for plaintiff » defendant appealed » Circuit Court affirmed » defendant
sought
writ of certiorari from Supreme Court » Supreme Court granted

*Note on Diversity SMJ* » how did we get into federal court in NY if accident took place in
PA?
Plaintiff (resident of NY) sues defendant train (PPB/incorporation = PA or somewhere other
than NY)
Defendant has substantial contacts with New York so she can be sued in NY or PA state court
Defendant can also be sued in NY or PA federal court because complete diversity exists
If you are in federal court in NY, how do you decide which law to apply?
You go to New York choice of law rules, which specifies the circumstances in which courts of
that state should follow laws of other jurisdictions [other states, federal or foreign law]
Inference from Erie » it was so obvious that NY would apply PA law so it was not addressed
If state law applies here, then what state law applies? Pennsylvania law
Why is New York law not applied here? [the court does not really address this]
Even though it is not explicitly addressed, implicit assumption is that New York [the place
where the case is brought] would apply Pennsylvania law because that is where everything
happened [New York choice of law]

RULE: In diversity actions in federal courts, only state substantive law will apply whether it be
statutes
made by the legislature or common law made by state courts and judicial opinions from the
highest court.
Swift doctrine is OVERTURNED.
REASONING: Judge makes two important points in reasoning why Swift must be overturned
(1) Statutory misinterpretation [the RDA] » Swift incorrectly construed state "laws" as used in
the
RDA to mean only statutory law made by the legislature
Correct interpretation of state "laws" = statutes + common law [judge-made law; judicial
opinions]
(2) *KEY REASON* Constitutional violation » Swift assumes unconstitutional power of federal
court
that permits federal courts/judges to ignore state common law in diversity cases
Bigger problem [than the statutory misinterpretation] = a foundational constitutional
problem.
U.S. Const. Article 1, §8 enumerates the powers of the federal courts; everything else is left
to the state courts » it does NOT give federal courts this independent power that is being
assumed under the Swift doctrine.

Downloaded From OutlineDepot.com

Concludes that Swift doctrine is an "unconstitutional assumption of powers" where federal


courts
have invaded Constitutional rights of states

HOLDING: Except in matters governed by the Federal Constitution or by Acts of Congress, the
law to be
applied in any case is the law of the state whether made by the legislature [statutes; statutory
law] or by the
highest court [common law; judge-made law; judicial opinions]
Whether the state law is statutory law or common law is not a matter of federal concern
There is no federal "general common law" thus, Congress has no power to declare
substantive rules of
common law applicable in a state whether rules are "local" or "general" and no clause in the
Constitution
confers such a power upon the federal courts
Rather, the constitution of the U.S. recognizes and preserves the autonomy and
independence of the
states [both in their legislative independence and judicial independence via their state judicial
departments/state courts] thus » any interference with state judicial action or state legislative
action
[except as permitted by the Constitution or some other reason] is an invasion of a state's
authority
and a denial of the state's independence.

TAKE-AWAY » Erie wanted to avoid Two (2) Problems created by Swift:


(1) Non-uniformity » common law issues answered differently by state and federal courts
(2) Forum Shopping » people choose state/federal court based on which is favorable to them
Black & White Taxicab v. Brown & Yellow Taxicab [taxicab company actually reincorporated
in another state just so they could get into federal court because then the federal court would
ignore any state law that was unfavorable to the corporation]
C. Guaranty Trust v. York [Erie S/P Test » Outcome-Determinative Test]
Erie doctrine » rule uses "formulaic" scientific terminology stating essentially that federal
courts
must follow state substantive law and federal procedural law
Court moves away from that terminology and the substance-procedure equation to a new
equation
Outcome-determinative test = If a state law is outcome determinative, then we must apply it
Does the state law, whether substantive or procedural [like the SOL rule in this case],
significantly affect the result of a litigation in a way that would result in different outcomes
depending on if it is tried in state court [where the law is controlling] or federal court [where
the
federal court disregards such state law]?
Outcome of litigation in federal court should be substantially the same as that in state court
thus » if the federal court disregards the state law, does it significantly affect the outcome?
If yes, then apply state rule no matter how it is labeled (substantive or procedural)
Goal = uniformity in federal/state outcomes when litigation outcomes depend upon whether
a legal
rule is enforced or not [i.e. SOL in York]
D. Byrd v. Blue Ridge [Application of York O-D Test to Judge/Jury Rules]
Step-by-Step Analysis:
(1) Is the state law [judge decides] “bound up with the rights and obligations created by the
state?”
(2) If no, then will applying the state law change the outcome of the case?
(3) Follow the state law unless there are countervailing considerations.
Federal interest here in having the 7th amendment followed is STRONG
REASONING: The state requirement [judge decides] is merely a form and mode of enforcing
the
immunity
Thus, it is not a rule intended to be bound up with the definition of the rights and obligations
of the
parties
Court found nothing to suggest that this rule was announced as an integral part of the special
relationship created by the statute
Court says question is: is the state created rule “bound up with state created rights and
obligations?”
If it is not, review the procedural interest in the integrity of the federal system in its
conflicting rule

Downloaded From OutlineDepot.com

7
th Amendment says there is a jury right in federal court in these types of cases.

HOLDING: Given that the state law of having the judge decide isn’t bound up with the rights
and
obligations of the state, then the countervailing rule of the federal system should apply
Court holds that right to trial by jury has implications tied up in the 7th Amendment of
Constitution, and
may not be outcome determinative, so apply federal law.
Take-Away Point: Outcome-Determinative test might not always be enough need to look at
other
countervailing considerations [the function of judge-jury relationship under 7th Amendment
which allocates
functions between judge and jury and assigns decisions of disputed questions of fact to the
jury]
When the likelihood of a different outcome is not so strong, then follow federal procedural
rules [i.e.
federal practice of jury determination of disputed factual issues] especially in light of the fact
that the
jury is assigned a function that is an essential factor in the process for which the Federal
Constitution
provides
E. Hanna v. Plumer [§2072: Rules Enabling Act » follow F.R.C.P. unless...]
ISSUE: Conflict between federal and state procedure rules on service of process (proper way
of giving
notice and serving summons and complaint on the defendant)
Federal Rule [4(e)(2)(b)]: service of process = leave with competent adult at defendant's
residence
State Rule: service of process = personal service of summons and complaint on defendant
York O-D Test » If state law applies, plaintiff must re-serve but by that time the SOL would
have run
Thus, the state rule would be outcome determinative because plaintiff wouldn't be able to
sue defendant
HOLDING: Despite state rule being outcome-determinative under York test, federal rule on
service of
process is within the scope of congressional mandate as conferred by the Rules of Enabling Act
[§2072] and
must be applied instead of the state rule.
Where does F.R.C.P get authority? Rules Enabling Act [28 U.S.C. §2072]
Supreme Court has the power to prescribe rules of practice and procedure in district courts.
[F.R.C.P.] Rules shall not abridge, enlarge or modify any substantive right [of state law]
(b) acts as a limitation on the federal procedural rules.
28 U.S.C. §2072. Rules of procedure and evidence; power to prescribe
(a) The Supreme Court shall have the power to prescribe general rules of practice and
procedure and rules of evidence for
cases in the United States district courts (including proceedings before magistrate judges
thereof) and courts of appeals.
(b) Such rules shall not abridge, enlarge or modify any substantive right. All laws in conflict with
such rules shall be of no
further force or effect after such rules have taken effect.
(c) Such rules may define when a ruling of a district court is final for the purposes of appeal
under section 1291of this title.
Example: F.R.C.P. Rule 4 [service of process]
Is Rule 4 abridging a state substantive right?
No » (1) it does not exceed the congressional mandate embodied in the REA and (2) it does
not
transgress constitutional bounds.

Hanna RULE: Follow federal procedure rule unless it affects some substantive right of state
law.
[Constitutional] REASONING » Federal courts do not have the power to affect or interfere
with
state substantive rights because the Constitution does not enumerate this power or authorize
such
power in the federal courts.
TAKE-AWAY: We follow FRCP rules unless they abridge, enlarge or modify substantive rights
of state
law
Back to Erie doctrine which says » follow state substantive law and follow federal procedural
law
Rationale of Hanna reflects Erie Goals: (1) Discourage forum-shopping; (2) Avoid non-
uniformity
Courts respond differently to same situation depending on procedure rule
Results in major problem that we want to avoid » inequitable administration of law
XII. PLEADINGS

Downloaded From OutlineDepot.com

A. Complaint
Rule 3. Commencing an Action.
A civil action is commenced by filing a complaint with the court
Rule 7. Pleadings Allowed; Form of Motions and Other Papers
(a) Pleadings. Only these pleadings are allowed:
(1) a complaint;
(2) an answer to a complaint;
(3) an answer to a counterclaim designated as a counterclaim;
(4) an answer to a crossclaim;
(5) a third-party complaint;
(6) an answer to a third-party complaint; and
(7) if the court orders one, a reply to an answer.
(b) Motions and Other Papers.
(1) In General. A request for a court order must be made by motion. The motion must:
(A) be in writing unless made during a hearing or trial;
(B) state with particularity the grounds for seeking the order; and
(C) state the relief sought.
(2) Form. The rules governing captions and other matters of form in pleadings apply to motions
and other papers
Rule 8. General Rules of Pleading
(a) Claim for Relief. A pleading that states a claim for relief must contain:
(1) a short and plain statement of the grounds for the court's jurisdiction, unless the court
already has jurisdiction and the claim needs no new jurisdictional support;
(2) a short and plain statement of the claim showing that the pleader is entitled to relief; and
(3) a demand for the relief sought, which may include relief in the alternative or different types
of relief.
(b) Defenses; Admissions and Denials.
(1) In General. In responding to a pleading, a party must:
(A) state in short and plain terms its defenses to each claim asserted against it; and
(B) admit or deny the allegations asserted against it by an opposing party.
(2) Denials--Responding to the Substance. A denial must fairly respond to the substance of the
allegation.
(3) General and Specific Denials. A party that intends in good faith to deny all the allegations of
a pleading--including the jurisdictional grounds--may do so by a
general denial. A party that does not intend to deny all the allegations must either specifically
deny designated allegations or generally deny all except those specifically
admitted.
(4) Denying Part of an Allegation. A party that intends in good faith to deny only part of an
allegation must admit the part that is true and deny the rest.
(5) Lacking Knowledge or Information. A party that lacks knowledge or information sufficient to
form a belief about the truth of an allegation must so state, and the
statement has the effect of a denial.
(6) Effect of Failing to Deny. An allegation--other than one relating to the amount of damages--
is admitted if a responsive pleading is required and the allegation is not
denied. If a responsive pleading is not required, an allegation is considered denied or avoided.
(c) Affirmative Defenses.
(1) In General. In responding to a pleading, a party must affirmatively state any avoidance or
affirmative defense, including:
• accord and satisfaction;
• arbitration and award;
• assumption of risk;
• contributory negligence;
• duress;
• estoppel;
• failure of consideration;
• fraud;
• illegality;
• injury by fellow servant;
• laches;
• license;
• payment;
• release;
• res judicata;
• statute of frauds;
• statute of limitations; and
• waiver.
(2) Mistaken Designation. If a party mistakenly designates a defense as a counterclaim, or a
counterclaim as a defense, the court must, if justice requires, treat the
pleading as though it were correctly designated, and may impose terms for doing so.
(d) Pleading to Be Concise and Direct; Alternative Statements; Inconsistency.
(1) In General. Each allegation must be simple, concise, and direct. No technical form is
required.
(2) Alternative Statements of a Claim or Defense. A party may set out 2 or more statements of a
claim or defense alternatively or hypothetically, either in a single
count or defense or in separate ones. If a party makes alternative statements, the pleading is
sufficient if any one of them is sufficient.
(3) Inconsistent Claims or Defenses. A party may state as many separate claims or defenses as it
has, regardless of consistency.
(e) Construing Pleadings. Pleadings must be construed so as to do justice.
Rule 10. Form of Pleadings
(a) Caption; Names of Parties. Every pleading must have a caption with the court's name, a title,
a file number, and a Rule 7(a) designation. The title of the complaint
must name all the parties; the title of other pleadings, after naming the first party on each side,
may refer generally to other parties.

Downloaded From OutlineDepot.com

(b) Paragraphs; Separate Statements. A party must state its claims or defenses in numbered
paragraphs, each limited as far as practicable to a single set of
circumstances. A later pleading may refer by number to a paragraph in an earlier pleading. If
doing so would promote clarity, each claim founded on a separate
transaction or occurrence--and each defense other than a denial--must be stated in a separate
count or defense.
(c) Adoption by Reference; Exhibits. A statement in a pleading may be adopted by reference
elsewhere in the same pleading or in any other pleading or motion. A
copy of a written instrument that is an exhibit to a pleading is a part of the pleading for all
purposes.
Rule 4(m): Time Limit for Service.
Defendant must be served within 120 days of a complaint being filed. Otherwise, the court
must dismiss (on motion or on its own) without prejudice or order
that service be made within a specified time. If the plaintiff shows a good cause for failure, the
court must extend the time for service for an appropriate
period. This rule does not apply to service in a foreign country under (f) or (j)(1).
Rule 7: (a) defines pleading [complaint, the answer and some other initial papers in a lawsuit];
and
(b) distinguishes pleading from motion [motions describe any request for a court order]
Rule 8(a). Claim for Relief. A pleading that states a claim for relief must contain:
(a) An allegation of jurisdiction;
(b) Short and plain statement of the claim showing the pleader is entitled to relief; and
(c) A demand for judgement for the relief sought, which may include relief in the alternative
or
different types of relief.
WHAT IS STANDARD OF PLEADING [FOR PLAINTIFF'S COMPLAINT]?
Old Rule [Conley v. Gibson] » Court says “a complaint should not be dismissed unless it
appears beyond doubt that the plaintiff can prove no set of facts in support of his claim which
would entitle him to relief.” If you made it conceivable that you could win
Complaint’s purpose » give fair notice to the defendant of what the claim is and the ground it
rests on and during discovery you find the specifics and details as to the facts will be
uncovered
Plaintiff just needs to include enough facts to show that his claim was conceivable
New Rule [Ashcroft v. Iqbal citing Bell Atlantic v. Twombley] » You need to include plausible
facts; your claim cannot be just conceivable, rather your claim must be PLAUSIBLE!
Standard of pleading [a complaint]: Conceivable [Conley] » Plausible [Twombly]
In order for a complaint to survive a motion to dismiss, the court will need to find that the
complaint contained sufficient factual matter [accepted as true] to state a claim to relief that
is plausible on its face
Facial plausibility standard » the plaintiff pleads factual content that allows the court to
draw the reasonable inference that the defendant is liable for the misconduct alleged
Plausibility is NOT probability but it asks for more than just sheer possibility

Two principles underlie Iqbal decision:


1. Recitals of elements of a cause of action + mere conclusory statements = BAD!
2. Only a complaint that states a plausible claim for relief will survive a motion to dismiss
This requires the Court to draw on its judicial experience and common sense because this is a
context-specific task [the task of determining whether a complaint states a plausible claim for
relief]
TAKE-AWAY:
What does plausibility mean?
When the plaintiff pleads factual content so that the court can make reasonable inference
that the
defendant did the unlawful act that the plaintiff asserts in the complaint
See Ashcroft v. Iqbal
Somewhere between the spectrum of possibility and probability
Possibility ----» (Plausibility) ----» Probability
What is the just piece of this equation? Justice » give defendant fair notice as to claim

Downloaded From OutlineDepot.com


Rule 9. Special Complaint. » [heightened pleading standard]
Rule 9: Pleading Special Matters.
(b) Fraud or Mistake; Conditions of Mind. In alleging fraud or mistake, a party must state with
particularity the circumstances constituting fraud or mistake. Malice,
intent, knowledge, and other conditions of a person's mind may be alleged generally.
Stradford v. Zurich [Plaintiff moves to dismiss defendants’ counterclaims based in fraud for
[failure
to state their claims with sufficient “particularity” under Rule 9(b) and failure to state a claim
with
respect to certain other counterclaims]
B. Sanctions [Rule 11.]
Rule 11(c) Sanctions.
(1) In General. If, after notice and a reasonable opportunity to respond, the court determines
that Rule 11(b) has been violated, the
court may impose an appropriate sanction on any attorney, law firm, or party that violated the
rule or is responsible for the violation.
Absent exceptional circumstances, a law firm must be held jointly responsible for a violation
committed by its partner, associate, or
employee.
(2) Motion for Sanctions. A motion for sanctions must be made separately from any other
motion and must describe the specific
conduct that allegedly violates Rule 11(b). The motion must be served under Rule 5, but it must
not be filed or be presented to the
court if the challenged paper, claim, defense, contention, or denial is withdrawn or
appropriately corrected within 21 days after service
or within another time the court sets. If warranted, the court may award to the prevailing party
the reasonable expenses, including
attorney's fees, incurred for the motion.
(3) On the Court's Initiative. On its own, the court may order an attorney, law firm, or party to
show cause why conduct specifically
described in the order has not violated Rule 11(b).
(4) Nature of a Sanction. A sanction imposed under this rule must be limited to what suffices to
deter repetition of the conduct or
comparable conduct by others similarly situated. The sanction may include nonmonetary
directives; an order to pay a penalty into
court; or, if imposed on motion and warranted for effective deterrence, an order directing
payment to the movant of part or all of the
reasonable attorney's fees and other expenses directly resulting from the violation.
(5) Limitations on Monetary Sanctions. The court must not impose a monetary sanction:
(A) against a represented party for violating Rule 11(b)(2); or
(B) on its own, unless it issued the show-cause order under Rule 11(c)(3) before voluntary
dismissal or settlement of the claims made
by or against the party that is, or whose attorneys are, to be sanctioned.
(6) Requirements for an Order. An order imposing a sanction must describe the sanctioned
conduct and explain the basis for the
sanction.
Walker v. Northwest Corp [HOLDING: It is the burden of plaintiff or his attorney to establish
diversity if
bringing claims on those grounds » If plaintiff or his attorney fails to meet this burden then
sanctions are
warranted to prevent this type of behavior]
Applicable Rules:
Rule 11(b): When you submit papers to the court you certify to the best of your ability and
knowledge that all you say is true and right
Rule 11(b)(2): Legal inaccuracy; legal contention of diversity = untrue
Plaintiff’s attorney improperly pled legal contention asserting SMJ on diversity grounds
Rule 12(b)(1): Motion to Dismiss [on Grounds of Lack of SMJ]
Christian v. Mattell [RULE: Under Rule 11, sanctions are limited to “papers” signed in violation
of the rule
[pleadings, written motions, and other papers that have been signed and filed in a given case,
11(a)]
11(a) does not authorize sanctions for other extrinsic things such as: discovery abuses or
misstatements
made to the court during an oral presentation

TIMELINE OF CASE: Where are we and where are we headed?


Dispute ---------------» Complaint [Rule 8(a); 9(b); “Twiqbal”] -----------» Response to Complaint
C. Responding to the Complaint » Defendant has Four (4) Options!
1. Ignore it; do nothing [default judgment]
2. Pre-Answer Motion [Rule 12: Defenses and Objections]

Downloaded From OutlineDepot.com

Rule 12(b) lists defenses that defendants are allowed to assert as a motion
All of them have a common feature » do not require the defendant to say whether
the plaintiff’s allegations are true
Rather, the defendant is pointing to some other defect that should cause the
court to dismiss the case before the defendant has to answer the plaintiff’s
complaint

12(b)(6) = demurrer » essentially says that the plaintiff’s story does not matter.
Asks the question: whether, assuming the pleaded facts are true, the law provides
any remedy. So this motion does two things:
Admits for the purposes of the motion, all the facts alleged in the complaint
Says that even if all the facts alleged are true, the law grants plaintiff no legal
remedy [fancy way of saying that the plaintiff’s story doesn’t matter legally]

Rule 12(b). How to Present Defenses.


Every defense to a claim for relief in any pleading must be asserted in the responsive pleading if
one is required. But a party may assert the following
defenses by motion (Motion to Dismiss):
1) Lack of SMJ;
2) Lack of PJ;
3) Improper venue;
4) Insufficient process;
5) Insufficient service of process;
6) Failure to state a claim upon which relief can be granted; and
7) Failure to join a party under Rule 19

3. Answer [Rule 8(b);(c)]


i. 8(c) » [affirmative defenses]
ii. 8(b)(4) » [admit; deny; not enough knowledge or information "NKI"

Rule 8(b) Defenses; Admissions and Denials.


(1) In General. In responding to a pleading, a party must:
(A) state in short and plain terms its defenses to each claim asserted against it; and
(B) admit or deny the allegations asserted against it by an opposing party.
(2) Denials--Responding to the Substance. A denial must fairly respond to the substance of the
allegation.
(3) General and Specific Denials. A party that intends in good faith to deny all the allegations of
a pleading--including the jurisdictional grounds--may do so by a
general denial. A party that does not intend to deny all the allegations must either specifically
deny designated allegations or generally deny all except those specifically
admitted.
(4) Denying Part of an Allegation. A party that intends in good faith to deny only part of an
allegation must admit the part that is true and deny the rest.
(5) Lacking Knowledge or Information. A party that lacks knowledge or information sufficient to
form a belief about the truth of an allegation must so state, and the
statement has the effect of a denial.
(6) Effect of Failing to Deny. An allegation--other than one relating to the amount of damages--
is admitted if a responsive pleading is required and the allegation is not
denied. If a responsive pleading is not required, an allegation is considered denied or avoided.
Rule 8(c) Affirmative Defenses.
(1) In General. In responding to a pleading, a party must affirmatively state any avoidance or
affirmative defense, including:
• accord and satisfaction;
• arbitration and award;
• assumption of risk;
• contributory negligence;
• duress;
• estoppel;
• failure of consideration;
• fraud;
• illegality;
• injury by fellow servant;
• laches;
• license;
• payment;
• release;
• res judicata;
• statute of frauds;
• statute of limitations; and
• waiver.
Zielinski v. Philadelphia Piers, Inc. [Doctrine of equitable estoppel applied to prevent a party
from taking
advantage of the statute of limitations where the plaintiff has been misled by conduct of such
party]

Downloaded From OutlineDepot.com

Defendants do not have a right to foster the plaintiff’s attorney’s error when he knew that
such error was
made and fostered such error through acts of omission [referring to the defendant’s failure to
make
accurate statements in the record, which would have deprived the plaintiff of his right of
action]
TAKE-AWAY: If you do not properly deny » you admit!
Defendant did not properly deny » rules are if you did not properly deny, you admit to
allegations!
Pretrial order is going to say that the defendant admitted to the plaintiff’s allegations
Why is the Court is submitting to the jury something they know is false?
Justice to the plaintiff is more important than the truth [JUSTICE > TRUTH]
Rule 8(e) Construing Pleadings » Pleadings must be construed so as to do justice.

HOW DO I ANSWER THE PLAINTIFF'S COMPLAINT?


Answers » look to the Rule 8(b)(1)
Options when it comes to answers:
Include affirmative defenses; and
Admissions or denials
Rule 8(b)(2): Denials » Responding to the Substance
Rule 8(b)(3): General and Specific Denials.
Rule 8(b)(4): Denying part of an allegation.
Rule 8(b)(5): Lacking Knowledge or Information
*DKI* » don’t know and don’t have information
Don’t admit or deny because WE DON’T KNOW » don’t have enough knowledge or
information to
admit or deny whether the allegation is true
If you don’t respond to the allegation, the court will assume you admit it
Rule 8(b)(6)
When plaintiff claims certain damages, you don’t need to respond
Rule 8(b)(6)
Failure to deny = ADMISSION
One always denies a damage statement just to be safe, but you do not have to do that
3 options for Response to Factual Allegations [in plaintiff’s complaint]:
1. Deny
2. Admit
3. DKI
D. Amendments [Rule 15. Amendment of Pleadings]

i. 15(c) » "relates back"

Rule 15. Amended and Supplemental Pleadings.


(a) Amendments Before Trial.
(1) Amending as a Matter of Course. A party may amend its pleading once as a matter of course
within:
(A) 21 days after serving it, or
(B) if the pleading is one to which a responsive pleading is required, 21 days after service of a
responsive pleading or 21 days after service of a motion under Rule
12(b), (e), or (f), whichever is earlier.
(2) Other Amendments. In all other cases, a party may amend its pleading only with the
opposing party's written consent or the court's leave. The court should freely
give leave when justice so requires.
(3) Time to Respond. Unless the court orders otherwise, any required response to an amended
pleading must be made within the time remaining to respond to the
original pleading or within 14 days after service of the amended pleading, whichever is later.
(c) Relation Back of Amendments.
(1) When an Amendment Relates Back. An amendment to a pleading relates back to the date of
the original pleading when:
(A) the law that provides the applicable statute of limitations allows relation back;
(B) the amendment asserts a claim or defense that arose out of the conduct, transaction, or
occurrence set out--or attempted to be set out--in the original pleading; or
(C) the amendment changes the party or the naming of the party against whom a claim is
asserted
If Rule 15(c)(1)(B) is satisfied and if, within the period provided by Rule 4(m) for serving the
summons and complaint, the party to be brought in by amendment:
(i) received such notice of the action that it will not be prejudiced in defending on the merits;
and
(ii) knew or should have known that the action would have been brought against it, but for a
mistake concerning the proper party's identity.
Beeck v. Aquaslide [ISSUE: Should we allow amendment of the pleading under the “all other
amendments”
aspect of 15(a)(2) when way outside of 21-day provision]

Downloaded From OutlineDepot.com


[Foman] Criteria to determine if justice requires amendment:
Is there any type of bad faith by moving party? (Honest mistake vs. knew counterfeit slides
out
there)
Was there undue delay? (No, just found out new facts)
Futility of the amendment? (Won’t fix anything so what’s the point?)
Prejudice to the nonmoving party?
Plaintiff could potentially bring a fraud claim; or
Amended complaint (against actual manufacturer) may relate back in order to get around SOL
Moore v. Baker [Amendments that relate back to the original complaint » exception to SOL
bar]
15(c)(2) » SOL bars complaint unless amendment relates back to the date of the original
complaint
Relates back to original date when the amendment asserts a claim or defense that arose out
of the
conduct, transaction, or occurrence set out- or attempted to be set out- in the original pleading
Critical issue in 15(c) determinations:
Whether the original complaint gave notice to the defendant of the claim now being asserted
[after
being amended]

Bonerb v. Caron Foundation [RULE: A claim relates back to the initial pleading when the claim
asserted in
the amended pleading arose out of the same nucleus of operative facts set forth in the original
pleading

While leave to amend a complaint should be freely given, an amendment which seeks to add
a time-
barred claim should not be allowed unless the otherwise untimely claim relates back to the
date of the

original pleading.

XIII.DISCOVERY
A. Initial Disclosures
i. Rule 26(a) » Required Disclosures

Rule 26. Duty to Disclose; General Provisions Governing Discovery.


(a) Required Disclosures.
(1) Initial Disclosure.
(A) In General. Except as exempted by Rule 26(a)(1)(B) or as otherwise stipulated or ordered by
the court, a party must, without awaiting a discovery request, provide
to the other parties:
(i) the name and, if known, the address and telephone number of each individual likely to have
discoverable information--along with the subjects of that information--
that the disclosing party may use to support its claims or defenses, unless the use would be
solely for impeachment;
(ii) a copy--or a description by category and location--of all documents, electronically stored
information, and tangible things that the disclosing party has in its
possession, custody, or control and may use to support its claims or defenses, unless the use
would be solely for impeachment;
(iii) a computation of each category of damages claimed by the disclosing party--who must also
make available for inspection and copying as under Rule 34 the
documents or other evidentiary material, unless privileged or protected from disclosure, on
which each computation is based, including materials bearing on the nature
and extent of injuries suffered; and
(iv) for inspection and copying as under Rule 34, any insurance agreement under which an
insurance business may be liable to satisfy all or part of a possible judgment
in the action or to indemnify or reimburse for payments made to satisfy the judgment.
B. General Discovery
i. Rule 26(b) » Request Relevant Disclosures

Rule 26(b) Discovery Scope and Limits.


(1) Scope in General. Unless otherwise limited by court order, the scope of discovery is as
follows: Parties may obtain discovery regarding any nonprivileged matter
that is relevant to any party's claim or defense and proportional to the needs of the case,
considering the importance of the issues at stake in the action, the amount in
controversy, the parties' relative access to relevant information, the parties' resources, the
importance of the discovery in resolving the issues, and whether the burden or
expense of the proposed discovery outweighs its likely benefit. Information within this scope of
discovery need not be admissible in evidence to be discoverable.
(2) Limitations on Frequency and Extent.
(A) When Permitted. By order, the court may alter the limits in these rules on the number of
depositions and interrogatories or on the length of depositions under Rule
30. By order or local rule, the court may also limit the number of requests under Rule 36.
(C) When Required. On motion or on its own, the court must limit the frequency or extent of
discovery otherwise allowed by these rules or by local rule if it determines
that:
(i) the discovery sought is unreasonably cumulative or duplicative, or can be obtained from
some other source that is more convenient, less burdensome, or less
expensive;
(ii) the party seeking discovery has had ample opportunity to obtain the information by
discovery in the action; or
(iii) the proposed discovery is outside the scope permitted by Rule 26(b)(1).
(3) Trial Preparation: Materials.

Downloaded From OutlineDepot.com

(A) Documents and Tangible Things. Ordinarily, a party may not discover documents and
tangible things that are prepared in anticipation of litigation or for trial by or
for another party or its representative (including the other party's attorney, consultant, surety,
indemnitor, insurer, or agent). But, subject to Rule 26(b)(4), those
materials may be discovered if:
(i) they are otherwise discoverable under Rule 26(b)(1); and
(ii) the party shows that it has substantial need for the materials to prepare its case and cannot,
without undue hardship, obtain their substantial equivalent by other
means.
(B) Protection Against Disclosure. If the court orders discovery of those materials, it must
protect against disclosure of the mental impressions, conclusions, opinions,
or legal theories of a party's attorney or other representative concerning the litigation.
(C) Previous Statement. Any party or other person may, on request and without the required
showing, obtain the person's own previous statement about the action or its
subject matter. If the request is refused, the person may move for a court order, and Rule
37(a)(5) applies to the award of expenses. A previous statement is either:
(i) a written statement that the person has signed or otherwise adopted or approved; or
(ii) a contemporaneous stenographic, mechanical, electrical, or other recording--or a
transcription of it--that recites substantially verbatim the person's oral statement.
(4) Trial Preparation: Experts.
(A) Deposition of an Expert Who May Testify. A party may depose any person who has been
identified as an expert whose opinions may be presented at trial. If Rule
26(a)(2)(B) requires a report from the expert, the deposition may be conducted only after the
report is provided.
(B) Trial-Preparation Protection for Draft Reports or Disclosures. Rules 26(b)(3)(A) and (B)
protect drafts of any report or disclosure required under Rule 26(a)(2),
regardless of the form in which the draft is recorded.
(C) Trial-Preparation Protection for Communications Between a Party's Attorney and Expert
Witnesses. Rules 26(b)(3)(A) and (B) protect communications between
the party's attorney and any witness required to provide a report under Rule 26(a)(2)(B),
regardless of the form of the communications, except to the extent that the
communications:
(i) relate to compensation for the expert's study or testimony;
(ii) identify facts or data that the party's attorney provided and that the expert considered in
forming the opinions to be expressed; or
(iii) identify assumptions that the party's attorney provided and that the expert relied on in
forming the opinions to be expressed.
(D) Expert Employed Only for Trial Preparation. Ordinarily, a party may not, by interrogatories
or deposition, discover facts known or opinions held by an expert who
has been retained or specially employed by another party in anticipation of litigation or to
prepare for trial and who is not expected to be called as a witness at trial. But
a party may do so only:
(i) as provided in Rule 35(b); or
(ii) on showing exceptional circumstances under which it is impracticable for the party to obtain
facts or opinions on the same subject by other means.
(E) Payment. Unless manifest injustice would result, the court must require that the party
seeking discovery:
(i) pay the expert a reasonable fee for time spent in responding to discovery under Rule
26(b)(4)(A) or (D); and
(ii) for discovery under (D), also pay the other party a fair portion of the fees and expenses it
reasonably incurred in obtaining the expert's facts and opinions.
(5) Claiming Privilege or Protecting Trial-Preparation Materials.

(A) Information Withheld. When a party withholds information otherwise discoverable by


claiming that the information is privileged or subject to protection as trial-
preparation material, the party must:

(i) expressly make the claim; and


(ii) describe the nature of the documents, communications, or tangible things not produced or
disclosed--and do so in a manner that, without revealing information itself
privileged or protected, will enable other parties to assess the claim.
(B) Information Produced. If information produced in discovery is subject to a claim of privilege
or of protection as trial-preparation material, the party making the
claim may notify any party that received the information of the claim and the basis for it. After
being notified, a party must promptly return, sequester, or destroy the
specified information and any copies it has; must not use or disclose the information until the
claim is resolved; must take reasonable steps to retrieve the information if
the party disclosed it before being notified; and may promptly present the information to the
court under seal for a determination of the claim. The producing party must
preserve the information until the claim is resolved.
Favale v. Roman Catholic Diocese of Bridgeport
RULE: A party must demonstrate that requested information is relevant to a claim or defense
in order to
prevail on a motion to compel.
Question asks whether information [plaintiff compels] is pertinent in relation to claim or
defense at issue
RELEVANCE is RELATIONAL » Would the information sought in the case help the party seeking
it prove or defeat the claim in question?
ii. Proportionality and Privacy

a) Rule 26(c) » Court can limit discovery to protect a party or person from
annoyance, embarrassment, or oppression

Rule 26(c) Protective Orders.


(1) In General. A party or any person from whom discovery is sought may move for a protective
order in the court where the action is pending -- or as an alternative on
matters relating to a deposition, in the court for the district where the deposition will be taken.
The motion must include a certification that the movant has in good faith
conferred or attempted to confer with other affected parties in an effort to resolve the dispute
without court action. The court may, for good cause, issue an order to
protect a party or person from annoyance, embarrassment, oppression, or undue burden or
expense, including one or more of the following:
(A) forbidding the disclosure or discovery;
(B) specifying terms, including time and place or the allocation of expenses, for the disclosure
or discovery;
(C) prescribing a discovery method other than the one selected by the party seeking discovery;
(D) forbidding inquiry into certain matters, or limiting the scope of disclosure or discovery to
certain matters;
(E) designating the persons who may be present while the discovery is conducted;
(F) requiring that a deposition be sealed and opened only on court order;
(G) requiring that a trade secret or other confidential research, development, or commercial
information not be revealed or be revealed only in a specified way; and
(H) requiring that the parties simultaneously file specified documents or information in sealed
envelopes, to be opened as the court directs.
(2) Ordering Discovery. If a motion for a protective order is wholly or partly denied, the court
may, on just terms, order that any party or person provide or permit
discovery.
(3) Awarding Expenses. Rule 37(a)(5) applies to the award of expenses.

Downloaded From OutlineDepot.com

Price v. Leflore [RULE: Under Rule 26, discovery may be limited when the burden or expense
of the
proposed discovery outweighs its likely benefit. Rule 26 provides a non-exclusive list of factors
to be
considered, which include the needs of the case, the amount in controversy, the parties’
resources, the
importance of the issues at stake in the action, and the importance of the discovery in resolving
the issues.
Under 26(b)(2)(C), discovery may be limited when the burden or expense of the proposed
discovery
outweighs its likely benefit; Court may limit discovery when the burden or expense of the
proposed
discovery outweighs its likely benefits. [26(b)(2)(C)(i)-(iii)]
Rengifo v. Erevos [ISSUE: immigration status inquiry]
RULE: Under 26(c), a protective order may be issued to prevent certain matters from being
inquired
into to protect a party or person from annoyance, embarrassment, oppression, or undue
burden or
expense.
Generally, courts have recognized the oppressive effect of inquiring into a party’s immigration
status
and authorization to work when not relevant to a material claim. This inquiry could lead to
intimidation that would inhibit plaintiffs from pursing their rights. Liu v. Donna Karan
International,
Inc.
C. Mechanisms for Discovery
i. Rule 33 » Interrogatories
ii. Rule 30 » Depositions
iii. Rule 34 » Document Discovery [Request for Production of Documents]
iv. Rule 36 » Request for Admissions
v. Rule 35 » Physical and/or Mental Examinations
D. Experts
i. Testifying Experts v. Non-testifying (Consulting) Experts

a) Testifying Experts:
1. Must disclose 90 days before [26(a)(2)(D)]
2. Must disclose report of expert
3. Testifying Experts must be hired to issue facts/opinions in prep for trial
b) Consulting Experts:
1. Do not have to disclose anything

ii. Rule 26(a)(2)

Rule 26. Duty to Disclose; General Provisions Governing Discovery.


(a) Required Disclosures.
(2) Disclosure of Expert Testimony.
(A) In General. In addition to the disclosures required by Rule 26(a)(1), a party must disclose to
the other parties the identity of any witness it may use at trial to present
evidence under Federal Rule of Evidence 702, 703, or 705.
(B) Witnesses Who Must Provide a Written Report. Unless otherwise stipulated or ordered by
the court, this disclosure must be accompanied by a written report--
prepared and signed by the witness--if the witness is one retained or specially employed to
provide expert testimony in the case or one whose duties as the party's
employee regularly involve giving expert testimony. The report must contain:
(i) a complete statement of all opinions the witness will express and the basis and reasons for
them;
(ii) the facts or data considered by the witness in forming them;
(iii) any exhibits that will be used to summarize or support them;
(iv) the witness's qualifications, including a list of all publications authored in the previous 10
years;
(v) a list of all other cases in which, during the previous 4 years, the witness testified as an
expert at trial or by deposition; and
(vi) a statement of the compensation to be paid for the study and testimony in the case.
(C) Witnesses Who Do Not Provide a Written Report. Unless otherwise stipulated or ordered by
the court, if the witness is not required to provide a written report, this
disclosure must state:
(i) the subject matter on which the witness is expected to present evidence under Federal Rule
of Evidence 702, 703, or 705; and
(ii) a summary of the facts and opinions to which the witness is expected to testify.
(D) Time to Disclose Expert Testimony. A party must make these disclosures at the times and in
the sequence that the court orders. Absent a stipulation or a court order,
the disclosures must be made:
(i) at least 90 days before the date set for trial or for the case to be ready for trial; or
(ii) if the evidence is intended solely to contradict or rebut evidence on the same subject matter
identified by another party under Rule 26(a)(2)(B) or (C), within 30 days
after the other party's disclosure.
(E) Supplementing the Disclosure. The parties must supplement these disclosures when
required under Rule 26(e).

Downloaded From OutlineDepot.com


iii. Rule 26(b)(4). Trial Preparation: Experts.

a) (D) says ordinarily you cannot depose or get disclosures from non-testifying
experts EXCEPT under (b)(4)(D)(ii) » showing exceptional circumstances that
makes it impractical for party to get facts or opinions any other way
[Example: 3 experts in the world; they all died; no other way to get
comparable information]

Thompson v. Haskell [FACTS: Dr. Lucas = consulting expert so none of his reports would be
discoverable
EXCEPT if there no other means for the defendant to get such info about the plaintiff’s mental
state 10 days
after she was fired by the defendant employer] [HOLDING: defendant’s request falls under the
exception
to the usual (b)(4)(D) rule]
Chiquita v. Reefer [RULE: A party may not discover facts known or opinions held by an expert
who has
been retained or specially employed by another party in anticipation of litigation or preparation
for trial and
who is not expected to be called as a witness at trial except upon a showing of exceptional
circumstances
under which it is impracticable for the party seeking discovery to obtain facts or opinions on the
same
subject by other means]
STEPS FOR ANALYZING DISCOVERY OF EXPERTS:
1. What information is Defendant seeking?
2. Is information being sought relevant?
3. Is information from an expert at all?
4. If an expert... What kind of expert?
Consulting because plaintiff is not using report in her case
5. Do exceptions exist to make consulting witness’s information discoverable?
Yes, because couldn’t get information by any other means.
This step is necessary because information from consulting expert witness usually is not
discoverable under Rule 26(b)(4), however, there is an exception to this rule » need to figure
out if exception applies to case at hand.

** On an exam, say this » EXCEPTIONAL CIRCUMSTANCES COULD EXIST HERE, HOWEVER, WE


DO NOT KNOW ENOUGH INFORMATION TO MAKE A DEFINITIVE CONCLUSION. [Key Point »
recognize that exceptional circumstances might exist under 26(b)(4)(D)(ii) » it is okay that you
do not know
enough information to come up with a complete answer] **
E. Work Product Rules [Rule 26]

i. Idea behind this rule » certain things that are done in preparation of litigation are
protected
from disclosure]
a) Privilege = communications [attorney-client emails, letters, etc.]
b) Work Product = trial prep [different than privilege; not attorney-client privilege]

ii. Privilege and Work-Product Rules » 26(b)(3)

a) Fact Work-Product » (b)(3)(A) [cannot get things that are done in the
preparation of litigation unless (1) you show you have a substantial need for those
things; and (2) it would cause you undue hardship or undue burden to get those things
any other way]
b) Opinion Work-Product » (b)(3)(B) [if you meet the exception from fact rule
(above) » to the extent that the attorney has written notes or impressions, then those
aspects of the notes must be redacted; just the facts are sent over, not thoughts, mental
ideas, impressions, etc.; redaction = opinion work-product]

Downloaded From OutlineDepot.com

c) Previous Statement Work-Product Rule » (b)(3)(C) [previous statements are


discoverable for any party or any other statement; you, yourself, are allowed to get a
copy of any statement that you made; you can be a party or nonparty to the suit]
[Example: you made statement to police, so you’re entitled to get a copy]

Hickman v. Taylor [RULE: Production of witnesses might be discoverable only when witnesses
are no
longer available or can only be reached with difficulty]
Rule 26 (b) Discovery Scope and Limits.
(3) Trial Preparation: Materials.
(A) Documents and Tangible Things. Ordinarily, a party may not discover documents and
tangible things that are prepared in anticipation of litigation or for trial by or
for another party or its representative (including the other party's attorney, consultant, surety,
indemnitor, insurer, or agent). But, subject to Rule 26(b)(4), those
materials may be discovered if:
(i) they are otherwise discoverable under Rule 26(b)(1); and
(ii) the party shows that it has substantial need for the materials to prepare its case and cannot,
without undue hardship, obtain their substantial equivalent by other
means.
(B) Protection Against Disclosure. If the court orders discovery of those materials, it must
protect against disclosure of the mental impressions, conclusions, opinions,
or legal theories of a party's attorney or other representative concerning the litigation.
(C) Previous Statement. Any party or other person may, on request and without the required
showing, obtain the person's own previous statement about the action or its
subject matter. If the request is refused, the person may move for a court order, and Rule
37(a)(5) applies to the award of expenses. A previous statement is either:
(i) a written statement that the person has signed or otherwise adopted or approved; or
(ii) a contemporaneous stenographic, mechanical, electrical, or other recording--or a
transcription of it--that recites substantially verbatim the person's oral statement.
F. Discovery Sanctions
i. Rule 37 » Enforcing Discovery

a) Security National Bank v. Abbott Laboratories [RULE: Under 37(b), a court has
wide latitude in determining sanctions for discovery violations]

Rule 26. Duty to Disclose; General Provisions Governing Discovery.


(e) Supplementing Disclosures and Responses.
(1) In General. A party who has made a disclosure under Rule 26(a)--or who has responded to
an interrogatory, request for production, or request for admission--must
supplement or correct its disclosure or response:
(A) in a timely manner if the party learns that in some material respect the disclosure or
response is incomplete or incorrect, and if the additional or corrective
information has not otherwise been made known to the other parties during the discovery
process or in writing; or
(B) as ordered by the court.
Rule 30. Depositions by Oral Examination
(g) Failure to Attend a Deposition or Serve a Subpoena; Expenses. A party who, expecting a
deposition to be taken, attends in person or by an attorney may recover
reasonable expenses for attending, including attorney's fees, if the noticing party failed to:
(1) attend and proceed with the deposition; or
(2) serve a subpoena on a nonparty deponent, who consequently did not attend.
XIV. RESOLUTION WITHOUT A TRIAL
Two procedural devices that force the parties to engage and respond to each other:
1.) Default Judgment = designed to urge the defendant into action [Rule 55]
2.) Involuntary Dismissal = intended to keep the plaintiff from going to sleep at the litigation
switch [Rule 41(b)]
A. Default Judgments
i. Rule 55
Rule 55. Default; Default Judgment.
(a) Entering a Default. When a party against whom a judgment for affirmative relief is sought
has failed to plead or
otherwise defend, and that failure is shown by affidavit or otherwise, the clerk must enter the
party's default.
(b) Entering a Default Judgment.
(1) By the Clerk. If the plaintiff's claim is for a sum certain or a sum that can be made certain by
computation, the clerk--on
the plaintiff's request, with an affidavit showing the amount due--must enter judgment for that
amount and costs against a
defendant who has been defaulted for not appearing and who is neither a minor nor an
incompetent person.
(2) By the Court. In all other cases, the party must apply to the court for a default judgment. A
default judgment may be
entered against a minor or incompetent person only if represented by a general guardian,
conservator, or other like fiduciary
who has appeared. If the party against whom a default judgment is sought has appeared
personally or by a representative, that

Downloaded From OutlineDepot.com

party or its representative must be served with written notice of the application at least 7 days
before the hearing. The court
may conduct hearings or make referrals--preserving any federal statutory right to a jury trial--
when, to enter or effectuate
judgment, it needs to:
(A) conduct an accounting;
(B) determine the amount of damages;
(C) establish the truth of any allegation by evidence; or
(D) investigate any other matter.
(c) Setting Aside a Default or a Default Judgment. The court may set aside an entry of default
for good cause, and it may
set aside a final default judgment under Rule 60(b).

ii. Rule 60(b) » permits reopening of the case even after judgment is entered on a defaults for
“any reason that justifies relief”

Rule 60. Relief from a Judgment or Order.


(b) Grounds for Relief from a Final Judgment, Order, or Proceeding. On motion and just terms,
the court may relieve a party or
its legal representative from a final judgment, order, or proceeding for the following reasons:
(1) mistake, inadvertence, surprise, or excusable neglect;
(2) newly discovered evidence that, with reasonable diligence, could not have been discovered
in time to move for a new trial under
Rule 59(b);
(3) fraud (whether previously called intrinsic or extrinsic), misrepresentation, or misconduct by
an opposing party;
(4) the judgment is void;
(5) the judgment has been satisfied, released or discharged; it is based on an earlier judgment
that has been reversed or vacated; or
applying it prospectively is no longer equitable; or
(6) any other reason that justifies relief.
Peralta v. Heights Medical [RULE: A requirement of due process in any proceeding is notice
reasonably
calculated to notify interested parties of pending action and afford them the opportunity to
present their
objections.
ISSUE: Can default judgment be set aside for failure of service because party cannot show he
has a
meritorious defense to the claim
Reasoning: Yes, a fundamental requirement of due process in any proceeding is NOTICE
[notice is
important because it notifies parties of the pending action and thus, affords them the
opportunity to
present their objections.
The courts below held that defendant was not harmed by the entering of the default
judgment
because he could not show that he had a meritorious defense to the action
Court rejected this reasoning of the lower court reasoning that defendant (Peralta) was
harmed and it
could have impleaded the employee, negotiated a settlement, paid the debt, or sold the
property
himself to pay the debt [all of which were precluded because defendant had no notice of the
action]
Holding » failure of notice = violation of due process rights » grant of plaintiff’s MSJ is
REVERSED.
B. Involuntary Dismissal
i. Rule 16 » Scheduling Orders [tells us what the judge “may do”]
1. 16(b) says a judge “must issue a scheduling order” unless the case belongs to an
exempt category » scheduling order must limit the time to: join other parties; amend
the pleading; complete discovery; file motions and it may set other dates too

ii. What can a defendant do if the plaintiff ignores a scheduling order’s deadlines or fails to
move suit forward? » Rule 41(b)
2. 41(b) [and other similar state statutes] allow for involuntary dismissal if plaintiff
“fails to prosecute” » failure to prosecute or some other grounds = reason for court to
permit involuntary dismissal

iii. Caussade v. United States [court involuntarily dismissed after plaintiff failed to make herself
available for a deposition, failed to fix mistakes in interrogatory responses, never made initial
disclosures, did not appear at pretrial conference, and did not keep in contact with her
attorney for months at a time]

Downloaded From OutlineDepot.com

C. Voluntary Dismissal » [Rule 41(a)]


Where do we get power to voluntarily dismiss a case? » Rule 41(a)
41(a)(1)(A)(i) » allows plaintiff to dismiss any time before defendant answers
Plaintiff can voluntarily dismiss at any time before defendant answered » after that, you need
either:
(1) Court’s permission [(a)(2)]; or (2) Agreement from the opposing party [(a)(1)(A)(ii)]
(a)(1)(A)(ii) allows plaintiff to dismiss a suit at any time if all the parties agree
(a)(2) authorizes a voluntary dismissal after defendant answers only by permission of the
court

i. Texaco v. Pennzoil [Facts: Plaintiff originally filed in Delaware » defendant opposed


injunction sought by plaintiff but did not answer » court denied plaintiff’s request for
preliminary injunction suggesting they would lose on merits of case » plaintiff quickly took
voluntary dismissal » refiled in Texas court » Texas jury eventually awarded plaintiffs $10
billion]
1. Take-Away Points: Since defendant did not answer Delaware complaint yet,
voluntary dismissal was available without court approval!
2. Defendant’s failure to answer » “one of the most expensive tactical errors ever
committed in a lawsuit”
3. Example of a case where the availability of a voluntary dismissal was catastrophic to
defendant
4. Advice for Lawyers: If a voluntary dismissal will harm your client » ANSWER
[PLAINTIFF’S] COMPLAINT PROMPTLY! Lock plaintiff into forum where suit
was brought before plaintiff dismisses and brings in new [more favorable] state
a. We [lawyers] want to AVOID this because if a court is favorable to the
plaintiff » likely to harm our clients

D. Settlement » WHY DO WE SETTLE?


Two (2) Reasons for Settlement:
Consensual Justice » What if a party simply runs out of funds to pursue the litigation?
“Unattractive” reason for settling
Idea behind this reason » A settlement [getting something] may be better than either one of
the
following options:
Getting nothing = happens if case gets dismissed; or --
Being stuck with a default judgment = happens if defendant is unable to pursue case
Cases collapse without regard to the merits of the case » this is much rarer today than it used
to be!
Some argue that settlements are not just faster and cheaper, but they are qualitatively better
because...
Consent is a basic principle of justice;
Settlements take into account other subtle facts/nuances; and
Settlements take into account the parties’ interests [that might be lost during adjudication].
Risk Control » What might the outcome of the lawsuit be [client asking lawyer]?
More common [thus, more important] reason for settling
Idea behind this reason »
Settlement controls risk
Trial outcomes have 2 characteristics:
(1.) Unpredictable [risky]
(2.) All-or-Nothing [expensive; catastrophic or a big loss to one party or sometimes both
parties]
For many parties, settlement = a way of avoiding an expensive and risky trial
i. HYPO CASE: Jane Smart v. GrowCo, Inc.

Downloaded From OutlineDepot.com


ii. Confidentiality in Settlement Agreements [Kalinauskas v. Wong]
1. RULE [Scope of Discovery » 26(b)(1)]: Parties may obtain discovery regarding any
unprivileged matter that is relevant to any party’s claim or defense.
2. REASONING: Although settlement agreement says it will impose penalties on (the
other plaintiff) Thomas if she discusses past employment with Caesars, court says
penalties won’t apply to the disclosure of information for discovery purposes in
furtherance of present case

i. Settlement agreement itself makes an exception for court ordered


release of information

3. TAKE-AWAY POINT: Policy Implications » settlement agreements that suppress


important evidence violates greater public policy concerns

XV. ADJUDICATION WITHOUT A TRIAL: SJ and MSJ » [Post-Discovery]

i. Two Basic Requirements for SJ:


a) No genuine issue as to material fact (no need for jury/fact finder); and
b) Movant is entitled to judgment as a matter of law (based on the facts presented,
the law favors one party over the other)

ii. Rule 56. Summary Judgment.


1. 56(e) » “the party opposing the MSJ bears the burden of responding only after the
moving party has met its burden of coming forward with proof of the absence of any
genuine issues of material fact”
2. 56(a) » mandates the entry of SJ after adequate time for discovery... against a party
who fails to make a showing sufficient to establish the existence of an element of that
party’s case [That party who also bears the burden of proof at trial]
3. 56(c)(1) » Moving party bears initial responsibility of informing the district court the
basis of its motion [MSJ]

1) (c)(1) » must identify the portion of parts of materials in the records


[such as depositions, interrogatories (like in this case)] that
demonstrate the absence of a genuine dispute
2) Moving party does not need to present evidence negating the
nonmoving’s party absence of evidence [Celotex]

Rule 56. Summary Judgment.


(a) Motion for Summary Judgment or Partial Summary Judgment. A party may move for
summary judgment, identifying each
claim or defense--or the part of each claim or defense--on which summary judgment is sought.
The court shall grant summary
judgment if the movant shows that there is no genuine dispute as to any material fact and the
movant is entitled to judgment as a
matter of law. The court should state on the record the reasons for granting or denying the
motion.
(b) Time to File a Motion. Unless a different time is set by local rule or the court orders
otherwise, a party may file a motion for
summary judgment at any time until 30 days after the close of all discovery.
(c) Procedures.
(1) Supporting Factual Positions. A party asserting that a fact cannot be or is genuinely disputed
must support the assertion by:
(A) citing to particular parts of materials in the record, including depositions, documents,
electronically stored information, affidavits
or declarations, stipulations (including those made for purposes of the motion only),
admissions, interrogatory answers, or other
materials; or
(B) showing that the materials cited do not establish the absence or presence of a genuine
dispute, or that an adverse party cannot
produce admissible evidence to support the fact.
(2) Objection That a Fact Is Not Supported by Admissible Evidence. A party may object that the
material cited to support or dispute a
fact cannot be presented in a form that would be admissible in evidence.
(3) Materials Not Cited. The court need consider only the cited materials, but it may consider
other materials in the record.
(4) Affidavits or Declarations. An affidavit or declaration used to support or oppose a motion
must be made on personal knowledge,
set out facts that would be admissible in evidence, and show that the affiant or declarant is
competent to testify on the matters stated.
(d) When Facts Are Unavailable to the Nonmovant. If a nonmovant shows by affidavit or
declaration that, for specified reasons, it
cannot present facts essential to justify its opposition, the court may:
(1) defer considering the motion or deny it;

Downloaded From OutlineDepot.com

(2) allow time to obtain affidavits or declarations or to take discovery; or


(3) issue any other appropriate order.
(e) Failing to Properly Support or Address a Fact. If a party fails to properly support an assertion
of fact or fails to properly address
another party's assertion of fact as required by Rule 56(c), the court may:
(1) give an opportunity to properly support or address the fact;
(2) consider the fact undisputed for purposes of the motion;
(3) grant summary judgment if the motion and supporting materials--including the facts
considered undisputed--show that the movant
is entitled to it; or
(4) issue any other appropriate order.
(f) Judgment Independent of the Motion. After giving notice and a reasonable time to respond,
the court may:
(1) grant summary judgment for a nonmovant;
(2) grant the motion on grounds not raised by a party; or
(3) consider summary judgment on its own after identifying for the parties material facts that
may not be genuinely in dispute.

iii. Celotex Corp v. Caltrett


1. HOLDING: Movant's burden = absence of evidence [lower standard] » found the
Adickes standard ridiculous; court held that it is the plaintiff's burden to present
evidence to prove that the defendant was liable AT TRIAL so the same burden should
apply at the MSJ stage]
i. Idea » align trial stage with MSJ stage via same burden
ii. If defendant can show plaintiff’s absence of evidence of her case then
MSJ will be granted » defendants do not have to say “here is evidence
of OUR [defendants] absence of liability”

2. Cites Adickes v. S.H. Kress & Co [holding that a to grant a SJ, moving party must
meet a higher burden » evidence of absence; said that the moving party (defendant)
has burden to show evidence of absence [of their liability]]

iv. Tolan v. Cotton


1. 56(c)(2) » Facts must be in admissible form [complies with evidence rules]
2. RULE: Must view evidence in light most favorable to the nonmoving party
3. HOLDING: Lower court improperly weighed evidence and facts in favor of moving
party [police officer defendant] which was wrong
i. Court held there was evidence that is in dispute that created a genuine
issue of material fact and thus, MSJ cannot be granted

v. Bias v. Advantage International


1. RULE: Once the moving party [defendant] has carried its burden, the responsibility
then shifts to the nonmoving party to show that there is, in fact, a genuine issue of
material fact.

1) “The nonmoving party must come forward with ‘specific facts’


showing that there is a genuine issue for trial."

2. HOLDING: Court says there is no genuine issue of material facts » Plaintiffs


“general evidence” [he is a good kid; six drug tests clean; coach thinks he is a good
guy] did not offset the defendant’s specific pieces of evidence about the victim's
times and dates of drug use

i. No rational trier of fact could find that the decedent was insurable
ii. Court's holding came down weighing moving party's evidence
[specific] and nonmoving party, the plaintiff's [general] evidence
3. What could the plaintiff have done in order to prevent grant of defendant's MSJ?

i. Poked hole in the defendant's evidence and version of facts


ii. Attacked teammate’s testimony » deposed these witnesses themselves
iii. Poked holes in terms of defendants' witnesses' credibility
iv. Submitted affidavits regarding the specific dates that the teammates
said they saw Bias using cocaine [provide alibi to discredit teammates
story]

Downloaded From OutlineDepot.com


a. Affidavits could have sworn that Bias was not doing cocaine
on such dates and times [See 56(c)(4)]

XVI. THE TRIER AND THE TRIAL


A. Judgment as a Matter of Law [Directed Verdicts]
i. Allows judge to make judgment at a matter of law when no reasonable jury could find
otherwise. This happens after discovery and opportunities for summary judgment and
dismissal
ii. Directed verdict happens after trial
iii. Trial happens » judge says no need for a jury [I’ll handle it from here]
iv. Judge is saying [just like with SJ] » no need for a jury because based on the law, a
reasonable jury could only reach one result
v. Evidence presented to jury » judge says I’m not going to let the jury rule on this case » I’m
going to direct a verdict
vi. Based on the law, a reasonable jury can only reach one outcome
vii. Rule 50(a). Judgement as a Matter of Law.

Rule 50. Judgment as a Matter of Law in a Jury Trial; Related Motion for a New Trial;
Conditional Ruling.
(a) Judgment as a Matter of Law.
(1) In General. If a party has been fully heard on an issue during a jury trial and the court finds
that a reasonable jury would not have
a legally sufficient evidentiary basis to find for the party on that issue, the court may:
(A) resolve the issue against the party; and
(B) grant a motion for judgment as a matter of law against the party on a claim or defense that,
under the controlling law, can be
maintained or defeated only with a favorable finding on that issue.
(2) Motion. A motion for judgment as a matter of law may be made at any time before the case
is submitted to the jury. The motion
must specify the judgment sought and the law and facts that entitle the movant to the
judgment.
THE DIRECTED VERDICT » [AFTER trial but BEFORE it goes to the jury]
In cases where a jury unanimously comes to an unsupportable conclusion in favor of a party,
the
opposing party can ask the judge to grant a “judgment as a matter of law.”
If the judge grants such motion, the case will not go to the jury.
It was called a “directed verdict” because...
The judge ordered [“directed”] then jury to return a verdict conforming to the evidence
When the jury does not return the requested [“directed”] verdict, a judge can grant a party’s
motion for a
directed verdict.
Reid v. San Pedro Railroad [RULE: In order to establish liability of the defendant, the plaintiff
must prove
liability by a preponderance of the evidence]
Further, when undisputed evidence of the plaintiff [giving rise to an inference of an essential
fact] points
to two things with equal weight [50/50] and only one of the things will render the defendant
liable, the
plaintiff must fail.
In such cases, the plaintiff will only prevail if she successfully showed [by a preponderance of
evidence] the facts and circumstances that would establish liability of the defendant

Penn Railroad v. Chamberlain » RULE: In cases where proven facts give equal support to each of
two
inconsistent inferences and the plaintiff failed to maintain the proposition [desired inference of
a fact] that
would establish the right to recover [and the defendant’s liability], then...
Judgment must be granted for the defendant because the plaintiff did not satisfy burden of
proof
[preponderance of the evidence] that would warrant the jury to make an inference of [sought
after]
essential fact from the evidence.
No evidence points directly to one fact over the other fact thus, jury cannot make a justifiable
inference that the plaintiff’s right to recover relies upon.

Downloaded From OutlineDepot.com

When does a case not go to the jury?


12(b) motion
MSJ
Directed verdict [jury is there in court room but jury actually never renders a verdict]
J.N.O.V. [judgment notwithstanding the verdict] » Standard it same as standard for MSJ and DV
B. Trial by Jury
A jury can sit in on a case when two conditions are met:
At least one party asks for a jury
It is a case of the sort which the parties are entitled to a jury
What gives you a right to a trial by jury? The Seventh Amendment [Bill of Rights in U.S.
Constitution]
7th Amendment does not create a right, but instead preserves a right that existed before the
Constitution
was enacted.
Only applies to federal courts because this is rule under the federal Constitution
State courts might apply rules under the respective state Constitution
Often, the Amendment is divided into two halves [both parts empower and limit the civil
jury]:
Clause #1: The “Right to a Jury” Clause [stating the right of trial by jury shall be preserved...]
Does not indicate the scope of the right being preserved
Clause #2: The “Reexamination” Clause [stating ...no fact tried by a jury, shall be otherwise
reexamined in any Court of the United States, then according to the rules of the common law.]

F.R.C.P. Rule 38 » reaffirms the constitutional status of jury trial


Insists on a timely demand and establishes waiver as a penalty for failing to do so

i. U.S. Constitution. 7th Amendment.

U.S. Const. Amend. VII. Civil Trial.


“In Suits at common law, where the value in controversy shall exceed twenty dollars, the right
of trial by jury shall be preserved, and no fact tried by a jury, shall be
otherwise reexamined in any Court of the United States, then according to the rules of the
common law.”

ii. Rule 38.


Rule 38. Right to a Jury Trial; Demand.
(a) Right Preserved. The right of trial by jury as declared by the Seventh Amendment to the
Constitution-- or as provided by a federal statute-- is preserved to the
parties inviolate.
(b) Demand. On any issue triable of right by a jury, a party may demand a jury trial by:
(1) serving the other parties with a written demand--which may be included in a pleading--no
later than 14 days after the last pleading directed to the issue is served;
and
(2) filing the demand in accordance with Rule 5(d).
(c) Specifying Issues. In its demand, a party may specify the issues that it wishes to have tried by
a jury; otherwise, it is considered to have demanded a jury trial on all
the issues so triable. If the party has demanded a jury trial on only some issues, any other party
may--within 14 days after being served with the demand or within a
shorter time ordered by the court--serve a demand for a jury trial on any other or all factual
issues triable by jury.
(d) Waiver; Withdrawal. A party waives a jury trial unless its demand is properly served and
filed. A proper demand may be withdrawn only if the parties consent.
C. Judgment Not Withstanding Verdict (JNOV)

i. Rule 50(b) » Same standard as directed verdict [50(a)]


Rule 50. Judgment as a Matter of Law in a Jury Trial; Related Motion for a New Trial;
Conditional Ruling.
(b) Renewing the Motion After Trial; Alternative Motion for a New Trial. If the court does not
grant a motion for judgment as a matter of law made under Rule
50(a), the court is considered to have submitted the action to the jury subject to the court's
later deciding the legal questions raised by the motion. No later than 28 days
after the entry of judgment--or if the motion addresses a jury issue not decided by a verdict, no
later than 28 days after the jury was discharged--the movant may file a
renewed motion for judgment as a matter of law and may include an alternative or joint
request for a new trial under Rule 59. In ruling on the renewed motion, the court
may:
(1) allow judgment on the verdict, if the jury returned a verdict;
(2) order a new trial; or
(3) direct the entry of judgment as a matter of law.

TIMELINE OF CASE and LITIGATION:

Downloaded From OutlineDepot.com

Plaintiff sues defendant » P makes 12(b) motion, court denies » discovery » defendant makes
MSJ, court denies
» defendant moves for DV » court denies [I’m sure the jury will find for the defendant anyway]
» trial happens
» judgment for the plaintiff » defendant moves for JNOV [for prior DV motion] » court really
wants to grant
JNOV [but it is a big deal to overturn jury’s verdict] » is there anything in between [not as harsh
in comparison
to grant of JNOV] that the court can do? YES! » grant and order a new trial
D. New Trials
Instead of granting harsh JNOV, court can order a new trial
Rule 59(d) » what are the grounds for granting or ordering a new trial?
“...may order a new trial for any reason that would justify granting one on a party's motion.”
i. Rule 59(d)
Rule 59. New Trial; Altering or Amending a Judgment.
(a) In General.
(1) Grounds for New Trial. The court may, on motion, grant a new trial on all or some of the
issues--and to any party--as follows:
(A) after a jury trial, for any reason for which a new trial has heretofore been granted in an
action at law in federal court; or
(B) after a nonjury trial, for any reason for which a rehearing has heretofore been granted in a
suit in equity in federal court.
(2) Further Action After a Nonjury Trial. After a nonjury trial, the court may, on motion for a
new trial, open the judgment if one has been entered, take additional
testimony, amend findings of fact and conclusions of law or make new ones, and direct the
entry of a new judgment.
(b) Time to File a Motion for a New Trial. A motion for a new trial must be filed no later than 28
days after the entry of judgment.
(c) Time to Serve Affidavits. When a motion for a new trial is based on affidavits, they must be
filed with the motion. The opposing party has 14 days after being
served to file opposing affidavits. The court may permit reply affidavits.
(d) New Trial on the Court's Initiative or for Reasons Not in the Motion. No later than 28 days
after the entry of judgment, the court, on its own, may order a new
trial for any reason that would justify granting one on a party's motion. After giving the parties
notice and an opportunity to be heard, the court may grant a timely
motion for a new trial for a reason not stated in the motion. In either event, the court must
specify the reasons in its order.
(e) Motion to Alter or Amend a Judgment. A motion to alter or amend a judgment must be filed
no later than 28 days after the entry of the judgment.
ii. Why do we have new trials? » (1) Flawed Procedures; and (2) Flawed Verdicts

Lind v. Schenley Industries [RULE: judge should not set aside the jury verdict as contrary to
the weight of
the evidence simply because he would have come to a different conclusion [if he were the trier
of the facts
and member of the jury].

FINAL TIMELINE: LITIGATION OF CASE

DISCOVERY » MSJ » IF MSJ DENIED » TRIAL » AT END OF TRIAL, CASE GOES TO JURY » UNLESS
DV IS GRANTED » IF DV IS GRANTED, CASE DOES NOT GO TO JURY » IF CASE DOES GO TO THE
JURY » AS A RESULT, JURY RENDERS A VERDICT » LOSER [BELOW] CAN ASK FOR JNOV [GIVE
ME VERDICT INSTEAD] AND/OR CNT » COURT CAN COME TO ONE OF A FEW DIFFERENT
OUTCOMES [SEE FOUR OUTCOMES LISTED ABOVE]

Você também pode gostar